Download as pdf or txt
Download as pdf or txt
You are on page 1of 109

DRAFT

KENYATTA UNIVERSITY
INSTITUTE OF OPEN LEARNING

SCH 302
CHEMISTRY OF AROMATIC COMPOUNDS

DR. A.W. WANYONYI


DR. A.K. MACHOCHO

CHEMISTRY DEPARTMENT
CHEMISTRY OF AROMATIC COMPOUNDS

PREFACE/GENERAL INTRODUCTION TO SCH 302

The course deals with benzene, its derivatives and other benzenoid compounds. Structure
of benzene aromaticity of benzene and related compounds will form the first part of the
course. Nomenclature of benzene and its derivatives are dealt with in details since this
will form some of the products of the reactions in the lessons that follow.

Mechanistic approach is employed to explain chemical reactions of benzene and its


derivatives. Predominant electrophilic aromatic substitution reactions patterns, effect of
substituent on the rates of reaction and orientation of incoming substituent are discussed.
Reactions of benzenoids are discussed in the light of similarity in chemical behavior to
benzene. It will be noted that many compounds will be synthesized in the course of
discussing these reactions as the end products

Key words in the relevant lessons and problems to enhance understanding or as set
induction to lesson to be covered next and give practice to students are provided in the
boxes. Each chapter has practice questions at the end and solutions are provided as an
illustration on how to approach similar problems.

Practicals in the appendix are meant to illustrate specific reactions in the actual
conditions applied in the preparation of aromatic compounds especially those covered in
the module.

ii
CONTENTS
PREFACE/GENERAL INTRODUCTION TO SCH 302............................................... ii
CONTENTS.................................................................................................................... iii
CHAPTER 1 .................................................................................................................... 1
STRUCTURE OF BENZENE AND AROMATICITY .................................................. 1
1.3 Characteristics of benzene ......................................................................................... 3
Heats of hydrogenation .................................................................................................... 4
Orbital picture of benzene................................................................................................ 5
1.4 Aromaticity and non-aromaticity............................................................................... 5
Questions........................................................................................................................ 13
CHAPTER 2 .................................................................................................................. 16
NOMENCLATURE OF SUBSTITUTED BENZENES AND OTHER ....................... 16
BENZENOID COMPOUNDS ...................................................................................... 16
Relative positions; 1:2, 1:3 and 1:4 on benzene ring are referred to as ortho (o), meta
(m) and para (p) respectively and sometimes used in nomenclature. ........................... 23
2.9 Questions and Solutions........................................................................................... 23
Questions........................................................................................................................ 23
Solutions ........................................................................................................................ 24
REACTIONS OF BENZENE........................................................................................ 25
Questions........................................................................................................................ 33
Solutions ........................................................................................................................ 33
CHAPTER 4 .................................................................................................................. 35
Questions........................................................................................................................ 44
Solutions ........................................................................................................................ 44
CHAPTER 5 .................................................................................................................. 47
ARENES AND ARYL HALIDES ................................................................................ 47
Questions........................................................................................................................ 55
Solutions ........................................................................................................................ 55
CHAPTER 6 .................................................................................................................. 57
PHENOLS...................................................................................................................... 57
Questions........................................................................................................................ 64
Solutions ........................................................................................................................ 65
CHAPTER 7 .................................................................................................................. 66
ANILINES ..................................................................................................................... 66
Solutions ........................................................................................................................ 72
CHAPTER 8 .................................................................................................................. 74
POLYNUCLEAR AROMATIC COMPOUNDS: NAPHTHALENE .......................... 74
Solutions ........................................................................................................................ 81
9.1.2 Diels Alder reaction .............................................................................................. 84
Questions........................................................................................................................ 88
REFERENCES .............................................................................................................. 90
EXPERIMENT 1: PREPARATION OF NITROBENZENE ........................................ 92
Introduction.................................................................................................................... 92
Questions........................................................................................................................ 93
EXPERIMENT 2: Reduction of nitrobenzene to Aniline.............................................. 94

iii
Questions........................................................................................................................ 95
EXPERIMENT 3: The nitration of Aniline to p-nitroaniline ........................................ 96
Introduction.................................................................................................................... 96
Procedure ....................................................................................................................... 96
Step 1: Preparation of acetanilide .................................................................................. 96
Step 2: Nitration of Acetanilide ..................................................................................... 96
Step 3: Hydrolysis of p-nitroacetanilide ........................................................................ 97
EXPERIMENT 4: Preparation of iodobenzene ............................................................. 98
Procedure ....................................................................................................................... 98
Step 1: Preparation of benzene diazonium chloride solution......................................... 98
Step 2: Preparation of iodobenzene ............................................................................... 98
[Note: Ether is very flammable] .................................................................................... 98
Questions........................................................................................................................ 99
Step 1: Diazotisation of sulphanilic acid ..................................................................... 100
Step 2: Preparation of methyl orange........................................................................... 101
Questions...................................................................................................................... 101
EXPERIMENT 6: Preparation of p- Di-t-butyl benzene............................................. 102
Introduction.................................................................................................................. 102
Procedure ..................................................................................................................... 102

Chapter 1: Structure of benzene and aromaticity


1.0 Introduction
1.1 Historical background on benzene
1.2 Structure of benzene
1.3 Characteristics of benzene ring
1.4 Aromaticity and non-aromaticity
1.5 Benzenoid aromatic and non- benzenoid compounds
1.6 Some of aromatic compounds in natural systems
1.7 Summary
1.8 Questions and solutions
1.9 Further reading

Chapter 2: Nomenclature of substituted benzenes and other


benzenoid compounds
2.0 Introduction
2.1 Nomenclature of monosubstituted benzene
2.2 Nomenclature of substituted benzene
2.3 Nomenclature of other benzene derivatives
2.4 Nomenclature of substituted naphthalenes
2.5 Nomenclature of anthracenes
2.6 Nomenclature of phenanthrenes
2.7 Summary
2.8 Questions and solutions
.
Chapter 3: Reactions of benzene
1.0 Introduction

iv
3.1 Reduction of benzene
3.2 Electrophilic aromatic substitution of benzene: A general mechanism
3.3 Halogenation
3.4 Nitration
3.5 Sulphonation
3.6 Friedel-Craft alkylation
3.7 Friedel–Craft acylation
3.8 Nitrosation
3.9 Summary
3.10 Questions and solutions

Chapter 4: Reactions of benzene derivatives


4.0 Introduction
4.1 Effect of substituents on EAS patterns/orientation
4.2 Groups that donate electrons to the ring
4.3 Effect of groups that donate by induction and release
electrons by resonance
4.4 Effect of a third substituent on the benzene ring
4.5 Summary
4.6 Questions and solutions

Chapter 5: Reactions of arenes and aryl halides


5.0 Introduction
5.1 Arenes
5.2 Reactions of Arenes
5.3 EAS reactions of aryl halide
5.4 Addition-elimination reactions of aryl halides
5.5 Removal of halide substituent from inactivated aryl halide
5.6 Summary
5.7 Questions and solutions

Chapter 6: Phenols
6.0 Introduction
6.1Physical and chemical properties of phenol
6.2 Laboratory preparation and industrial manufacture of phenol
6.3 Reactions of phenols
6.3.1 Ether and ester formation
6.3.2 Nitration and sulphonation of phenol
6.3.3 Bromination and acylation of phenol
6.3.4 Kolbe and coupling reactions of phenol
6.4 Uses of Phenols
6.5 Summary
6.6 Questions and solutions

Chapter 7: Anilines
7.0 Introduction

v
7.1 Preparation and properties of aniline
7.2 Reactions of aniline
7.3 preparation and reactions of diazonium ion
7.4 Summary
7.5 Questions and solutions

Chapter 8: Polynuclear aromatic compounds: Naphthalene


8.0 Introduction
8.1 Synthesis of naphthalene
8.2 Reduction-oxidation reactions of naphthalene
8.3 Orientation and EAS reactions of naphthalene
8.4 Electrophilic aromatic substitution reactions of naphthalene
8.5 Orientation EAS reactions naphthalenes derivatives
8.6 Summary
8.7 Questions and solutions

Chapter 9: Polynuclear aromatic compounds: Anthracene and Phenanthrene


9.0 Introduction
9.1 Reactions of anthracene and phenanthrene
9.1.1 Oxidation
9.1.2 Reduction and Diels-Alder reactions
9.1.2 Halogenation of anthracene and phenanthrene
9.1.3 Sulphonation of anthracene and phenanthrene
9.2 Alcohols of anthracene
9.3 Positional activity of anthracene
9.4 Anthraquinones
9.5 Summary
9.6 Questions and solutions

References and Further reading

vi
CHAPTER 1

STRUCTURE OF BENZENE AND AROMATICITY

1.0 Introduction
This chapter will look at detailed scientific arguments that let to structure establishment
of benzene, concept of sp2 orbitals and delocalisation of electrons in p orbitals forming pi
(π) electron cloud above and below benzene molecule. Application of Huckel’s rule to
benzenoid compounds to establish aromatic and non-aromatic compounds will be
discussed. Some important aromatic compounds in natural systems will be mentioned as
appreciation of the importance of aromatic compounds

Objectives

By the end of this lesson, you should be able to:

• Discuss the background and arguments that led to the establishment of benzene
structure
• Describe bonding in benzene molecule
• Explain using resonance, the stability of benzene ring
• State reasons why benzene is susceptible to electrophiles
• State the criteria for aromaticity
• Distinguish aromatic, benzenoid aromatic and non-aromatic compounds using
Huckel’s rule
• State some of the important aromatic compounds in natural systems

1.1 Historical background on benzene


Benzene was discovered by Michael Faraday in 1825 from whale oil and reported
empirical formula as CH2. However, the correct molecular formula was established in
1834 when Eihartd Mitscherlich synthesized it and was given as C6H6 and this formula
elicited many structure suggestions.

Compounds like benzene with relatively few hydrogens compared to carbons are usually
obtained from natural oils from either plants or animals. Due to their fragrance, they were
called aromatic to distinguish them from aliphatic, which have higher hydrogen to carbon
ratio obtained from degradation of fats. Figure 1 shows some of the most common
benzenoid compounds.

1
or

Benzene Naphthalene Anthracene

Phenanthrene
Pyrene Triphenylene
Figure 1. Some common benzenoid compounds

1.2 Structure of benzene


There were various suggestions to the structure of benzene that would fit the formula
C6H6. Some of such structures are illustrated in figure 2 below.

1. 2. 3. CH2

Kekule Dewar
Structure Structure

4. 5.

Figure 2. Some of the proposed structures of benzene

In the course of establishing the actual structure of benzene the following observations
were made:
1. Benzene has only one mono-substituted isomer, C6H5Y (Y = Cl, Br, I, NO2
etc). This argument eliminates structures 3 and 5.
2. Benzene yields only three (3) disubstituted isomers, C6H4XX or C6H4XY.
(e.g. C6H4Br2 or C6H4BrNO2). Structure 4 of the diyne is eliminated because it
would give two isomers. Structure 2 (Dewar) was eliminated because it would
give 6 structures!

X X
Y
X Y
Y
1,2 1,3 1,4

2
From these arguments only the Kekule structure remained.

He visualized benzene as a dynamic structure where the double bonds are delocalized (π
electrons are non-localized).

The bond lengths in benzene are the same of about ≈ 1.39Å. This is established with X-
ray diffraction techniques.
Ethane C-C ≈ 1.54 Å
Ethene C=C ≈ 1.34 Å

Thus the carbon-carbon bond in benzene is a hybrid of single and double bond.

1.3 Characteristics of benzene


It has been established that benzene has very different properties from closely related
molecules. A comparison of benzene with cyclohexene and cyclohexadiene is made.

Cyclohexene Cyclohexadiene Cyclohexatriene


(Benzene)

The alkenes undergo electrophilic addition reactions with H2, Cl2, Br2, I2 etc. Benzene on
the other hand prefers to undergo electrophilic substitution (EAS) reactions. The table
below illustrates some other reactions where benzene behaves differently from the other
two related compounds.

Reagent Cyclohexene/ Benzene


Cyclohexadiene
1. KMnO4 (cold, dilute) Rapid decolorization No reaction
2. Br2 in CCl4 Rapid decolorization No reaction
3. HI Rapid addition Reaction No reaction
4. H2 in presence of Ni Rapid reaction at 250C No reaction but slow reaction
and low pressures at high temperatures and
pressures

Thus benzene has some special stability.

3
Heats of hydrogenation

Consider the following heats of hydrogenation of benzene and related compounds.


Hydrogenation of the three forms cyclohexane.
H (Kcalmol-1)
H2, Heat
28.6

2H2, Heat
55.4

3H2, Heat
49.8

The information can be represented in form of energy diagram.

Cyclohexanetriene
(Calculated)

Cyclohexanediene

Benzene
Cyclohexene 36.0
85.8 Resonance
Energy Energy
(Kcalmol-1)
55.4 49.8
28.6

Progress of the Reaction

Figure 3. Energy diagram of benzene and related compounds

It is observed that the heat of hydrogenation of benzene is lower than expected by 36.0
Kcal mol-1. This is known as stabilization or resonance energy.

This is because the hydrogenation is an addition reaction that converts benzene into a less
stable structure as the aromaticity is lost. It is therefore, easier for benzene to undergo
substitution reactions where the aromatic properties are retained.

4
Orbital picture of benzene

All carbon atoms of benzene are sp2 hybridized. Therefore, it is a trigonal planer with
bond angles of about 1200. It has six π (pi) electrons (3 π bonds), which are delocalized
in the p-orbitals.

It is visualized as if there is a cloud of the π electrons above and below the plane of the
benzene molecule. The π electrons are loosely held than the δ (sigma) electrons and are
available for starting a reaction. Hence, benzene is susceptible to electrophilic reagents.

E
1 H
1
E Y
E+ H -
Y
+
2
2 E

Stable as the aromaticity


is retained

Question: Identify the type of reaction indicated by 1 and 2 above and comment on
the stability of the compounds.

1.4 Aromaticity and non-aromaticity


Aromatic compounds are those that resemble benzene in behavior. They must have the
following features:

1. Delocalized π electrons.
2. The delocalized π electrons that satisfy Huckel’s rule; 4n + 2; where n = 0, 1,
2, 3, 4 etc. That is, the total number of π electrons must 2, 6, 8, 10, 14, 18 etc.
3. Cyclic.
4. Planar.

Note: There are aromatic compounds that do not resemble benzene in terms of
structure and may not contain the phenyl ring.

5
The definition of aromatic compounds can therefore be modified to read: Aromatic
compounds are those cyclic planar compounds in which the π electrons energy of the
cyclic form is lower than that of the open chain.

1.5 Benzenoid aromatic and non-benzenoid compounds


Benzenoids are compounds, which resemble benzene in terms of structure and behavior.

Benzene Naphthalene

Benzene has six π electrons that are delocalized. It is planar and cyclic. 6 = 4n + 2 ⇒ n =
1. Therefore it is aromatic.
Naphthalene has 10 π electrons (5 double bonds) that are delocalized. It is planar and
cyclic. 10 = 4n + 2 ⇒ n = 2. It is aromatic.

Anthracene Naphthacene

Anthracene has three benzene rings fused in a linear manner. It has 14 π electrons (7
double bonds) that are delocalized. It is planar and cyclic. 14 = 4n + 2 ⇒ n = 3. It is
aromatic.
Naphthacene (Not naphthalene!) has 18 π electrons (9 double bonds) that are delocalized.
It is planar and cyclic. 18 = 4n + 2 ⇒ n = 4. It is aromatic.

Triphenylene

Triphenylene has 18 π electrons (9 double bonds) that are delocalized. It is planar and
cyclic. 18 = 4n + 2 ⇒ n = 4. It is aromatic.

6
1.5.1 Non-benzenoid aromatic compounds
These are compounds that are aromatic but are not benzenoid, that is, they do not
resemble benzene and do not contain the phenyl group.

-
-
-

The structures represent cyclopentadienyl anion. As shown the negative charge can be
delocalized to other positions in the structure. It has 6 π electrons (2 double bonds plus
one negative charge) and they are delocalisable, it is planar and cyclic. 6 = 4n + 2 ⇒ n =
1. It is aromatic. The anion forms stable complexes with transition metals because of
aromatic properties.

Consider cyclopentadienyl cation. Is it aromatic?

+
+
+

The positive charge can be delocalisable; the cation is planar and cyclic. It has 4 π
electrons; thus 4 = 4n + 2 ⇒ n = 0.5. This not an integer and therefore system does not
satisfy the Huckel’s rule. Thus cation is not aromatic.

Question: Explain why cyclopentadiene is a stronger acid than most other


hydrocarbons.

Answer: Acidity is loss of a proton by a given species (like a molecule).


Cyclopentadiene therefore dissociates into a hydrogen ion and cyclopentadienyl anion,
which has been described as stable because of being aromatic. Therefore the
dissociation is favored.

This is cycloheptatriene cation formed after loss of a proton. It has six π electrons
delocalisable electron (the positive charge be delocalized). It is planar and cyclic. 6 = 4n
+ 2 ⇒ n = 1. It is therefore aromatic. The corresponding anion is not aromatic because it
does not obey Huckel’s rule.

7
+

Cyclopropenyl cation has two π electrons delocalisable electron. It is planar and cyclic.
2 = 4n + 2 ⇒ n = 0. It is therefore aromatic. Its corresponding anion is not aromatic
because it does not obey Huckel’s rule.

Azulene

Azulene is a compound containing a seven membered and five membered rings fused
together. It has 10 π electrons delocalisable electron. It is planar and cyclic. 10 = 4n + 2
⇒ n = 2. It is therefore aromatic.

Note: Sometimes delocalized electrons do not have to be the π electrons but can be
non-bonding electrons (lone pairs of electrons).

This is the case in hetero aromatic compounds where the heteroatoms like O, N, S or P
are introduced in the ring. Their lone pairs of electrons are able to contribute to the
aromatic behavior. Some examples are sited below.

-
- N
HN
.. N H +
H +

Pyrole is a five-membered ring with nitrogen as the heteroatom. By utilization of the


lone pair of electrons of nitrogen, there are six delocalisable electrons. It is planar and
cyclic. 6 = 4n + 2 ⇒ n = 1. It is therefore aromatic.

:O: : S:

Furan and thiophene have oxygen and sulphur as the heteroatoms, respectively. By
utilization of one lone pair of electrons of oxygen or sulphur, the molecules have six
delocalisable electrons each and will obey Hucke’s rule.

N
Pyridine

8
Pyridine is planar cyclic and obeys Huckel’s rule. Note that the lone pair of electrons in
nitrogen note utilized in this case. It has similar properties as the benzenoids and
undergoes electrophilic aromatic substitution reactions.

1.5.2 Anti-aromatic compounds

These are compounds in which the cyclic forms have higher π electrons energy than the
open chain analogues. When the ring opens up what is formed is still anti-aromatic. The
best example representing this group of compounds is cyclobutadiene.

-H2

Lower Energy Higher Energy

. -H2 .

Lower Energy Higher Energy

-H2
+ +

Lower Energy Higher Energy

. .
-H2

Lower Energy Higher Energy

All the species do not obey Huckel’s rule and as indicated the cyclic forms are of higher
energy then their open analogues.

1.5.3 Benzenoid compounds that do not satisfy Huckel’s rule


There are some compounds that contain benzene rings fused in different ways but do not
satisfy the Huckel’s rule, the main characteristic feature of aromaticity, yet they are still
considered aromatic. Consider the following examples:

Pyrene 10b,10c-Dihydropyrene

9
Which of these two compounds is aromatic and which is not? Pyrene is cyclic, has
delocalisable electrons and planar. But it has 16 π electrons (8 double bonds) and
applying Huckel’s rule; 16 = 4n + 2 ⇒ n = 3.5, it does not satisfy it. However the
molecule is considered aromatic. 10b,10c-Dihydropyrene on the other hand has 14 π
electrons and satisfies Huckel’s rule and all other conditions of aromaticity. It is
considered as ‘fully aromatic’ in comparison with pyrene.

Benzo[def]chrysene 12b,12c-Dihydrobenzo[def]chrysene
(Benzo[a]pyrene) (12b,12c-Dihydrobenzo[a]pyrene)

Benzo[a]pyrene has 20 π electrons and will therefore not satisfy the Huckel’s rule. But it
is planar, has delocalisable electrons and cyclic. It is also considered as aromatic.
12b,12c-dihydrobenzo[a]pyrene with 18 π electrons is fully aromatic as it satisfies the
Huckel’s rule.

Note: Huckel’s rule strictly applies to mono-benzenoid compounds.

1.5.4 Annulenes
These are non-benzenoid multi cyclic compounds. Some of annulenes are not aromatic
yet they may be satisfying the Huckel’s rule. Consider [10]annulene for example.

Naphthalene Cyclodecapentaene
([10]annulene)

A closer look indicate that [10]annulene can be considered to be modified from


naphthalene whereby the bridging bond is absent. However, it remains with 10 π
electrons like naphthalene. It therefore satisfies Huckel’s rule. But it is not planar and not
considered as aromatic.

Anthracene Cyclotetradecaheptaene
([14]annulene)

10
When anthracene loses the two bridging bonds, it forms [14] annulene, which retains the
seven double bonds (14 π electrons). Like [10]annulene, [14]annulene is cyclic satisfies
the Huckel’s rule and the electrons are delocalisable. Unlike [10]annulene, however,
[14]annulene is planar and therefore aromatic.

Cyclooctatetraene
Bicyclo[4.2.0]octa-1(8),2,4,6-
([8]annulene)
tetraene

Both of the above compounds are not aromatic. Although [8]annulene has the electrons
delocalisable and cyclic, is not planar, does not satisfy Huckel’s rule. It is termed as anti-
aromatic because it is of higher π electrons energy than its open analogue, octa-1,3,5,7-
tetraene.

1.6 Some of aromatic compounds in natural systems


Aromatic compounds are of importance in natural systems. Many redox reactions in the
cell occur due presence of co-enzymes (complex compounds required for any enzyme to
function in a biological reactions), which are aromatic in nature. Some of the most
important amino acids are aromatic. These are phenylalanine and tyrosine.

O
O H2N CHC OH
H2N CHC OH CH2
CH2

OH
2-Amino-3-phenyl-propionic acid 2-Amino-3-(4-hydroxy-phenyl)-propionic acid
(Phenylalanine) (Tyrosine)

Structures of a number of hormones contain aromatic rings. A good example is


adrenaline, which acts as a transmitter of nerve impulses.

11
Ι
Ι O
NH2

HO Ι CO2H
Ι

2-Amino-3-[4-(4-hydroxy-3,5-diiodo-phenoxy)-3,5-diiodo-phenyl]-propionic acid
(Adrenaline)

Estrone (oestrone) is produced by mammalian ovaries and controls development of


female characteristics and menstruation cycle. As a drug, it is used for replacement
therapy in deficiency states like primary amenorrhoea (abnormal absence of
menstruation), delayed onset of puberty, control and management of menopausal
syndrome and malignant neoplasm of the prostate.

H H
HO
Estrone

Deoxyribonucleic acid (DNA) and Ribonucleic acid (RNA) are essential for storage of
genetic information and synthesis of enzymes and proteins needed for metabolisms. They
are composed of, among other chemical features, various aromatic bases of purine and
pyrimidine, which are arranged different sequence.

NH2 O
N N N NH
N N
N N N N NH2
N N H H
H
Adenine Guanine
Purine

NH2
O
N
N NH
N O
N H N O
H
Pyrimidine
Cytosine Thymine

Note that genetic information flows from DNA to RNA to proteins but in case of
retroviruses like HIV the information flows is reverse, that is, RNA to DNA to proteins.

12
Melanine, a polymer of indole derivative, is the dark pigment of the skin.

O
COOH
O N
H
HOOC
O
N
H O

Melanine

1.7 Summary
Benzene molecule has increased electron density due to delocalized electrons in sp2
orbitals.
Benzene is very stable due to high resonance/delocalisation energy.
Aromatic nature of benzene makes it to have unique chemical properties.
Anti-aromatic compounds have higher energies in cyclic form than in the corresponding
open chain analogues.
The conditions have to be satisfied by a compound before it is called aromatic.
Benzenoids are aromatic compounds that resemble benzene in terms of structure.
There are benzenoid compounds that do not satisfy Huckels rule but are aromatic.
Many redox reactions in cells occur due to co-enzymes that are aromatic in nature.

1.8 Questions and Solutions

Questions

Q1. (a) Draw all structures that will satisfy the formula C6H6.
(b) Assume that mono bromination substitutions were carried out in all the
structures in (a) above. Which of the structures will give isomers of the products?
Q2. Analyze and classify the following chemical species as aromatic, non-aromatic or
anti-aromatic.

(i) + (ii) - (iii)


N

(iv) (v) -
(vi)

(vii) (viii) - (ix) +

13
Solutions
1 (a)

1. 2. 3. CH2

Kekule Dewar
Structure Structure

4. 5.

(b) (i) Kekule structure(1) has only one product.


(ii) Dewar structure (2) has two products.
(iii) Structure 3 has three products.
(iv) Structure 4 has only one product
(v) Structure 5 has two products

Mono bromination Br

Mono bromination
+
Br
2 Br

Br Br
CH2 Mono bromination
CH2 + CH2 + CHBr
3
Mono bromination

Br
4
Br
Mono bromination
+
5 Br
2.
(i) Cycloheptatrienyl cation has 6 π electrons, hence, 6 = 4n +2, n = 1. Therefore
obeys Huckel’s rule. It is planar, cyclic and electrons are delocalisable. Thus it
is aromatic.
(ii) Cyclopropenyl anion has 4 π electrons, hence, 4 = 4n +2, n = 0.5. Therefore
does not obey Huckel’s rule. Thus it is non-aromatic.
(iii) Pyridine has 6 π electrons a lone pair on nitrogen which is not involved in
delocalisation in this case, hence, 6 = 4n +2, n = 1. Therefore it obeys

14
Huckel’s rule. It is planar, cyclic and electrons are delocalisable. Thus it is
aromatic.
(iv) Cyclobutadiene has 4 π electrons, hence, 4 = 4n +2, n = 0.5. It does not obey
Huckel’s rule. Its open chain analogue has lower π electron. Thus it is anti-
aromatic.
(v) Cyclopentadienyl anion has 6 π electrons, hence, 6 = 4n +2, n = 1. Therefore
it obeys Huckel’s rule. It is planar, cyclic and electrons are delocalisable. Thus
it is aromatic.
(vi) 9bH-Benzopyrene is a poly benzenoid compound. It has 18 π electrons, hence,
18 = 4n +2, n = 4. Therefore obeys Huckel’s rule. It is planar, cyclic and
electrons are delocalisable. Thus it is aromatic.
(vii) Cyclooctatetraene has 8 π electrons, hence, 8 = 4n +2, n = 1.5. It does not
obey Huckel’s rule. Its open chain analogue has lower π electron. Thus it is
anti-aromatic.
(viii) Cycloheptatrienyl anion has 8 π electrons, hence, 8 = 4n +2, n = 1.5. It does
not obey Huckel’s rule. Although its π electrons are delocalisable, its cyclic
and planar, it is not aromatic due to Huckel’s rule. It is termed as non-
aromatic.
(ix) Cyclopentadienyl cation has 4 π electrons, hence, 4 = 4n +2, n = 0.5. It does
not obey Huckel’s rule. Its open chain analogue has lower π electron. Thus it
is anti-aromatic.

15
CHAPTER 2

NOMENCLATURE OF SUBSTITUTED BENZENES AND OTHER


BENZENOID COMPOUNDS

2.0 Introduction
In chapter one, we looked at the structure of benzene and other benzenoid compounds. In
this chapter we are going to look at convectional rules to follow when naming
monosubstituted; disubstituted and polysubstituted benzene and other benzene related
compounds. IUPAC system of nomenclature will be employed in most cases. However,
special names that are internationally accepted will also be used.

Objectives

By the end of this lesson, you should be able to:

• State both IUPAC names and special names for monosubstituted benzene
• Draw structures when given names of monosubstituted benzene
• State both IUPAC names and special names for substituted benzene
• Draw structures when given names of substituted benzene
• State both IUPAC names and special names of other benzene derivatives
• Draw structures when given names of benzene derivatives
• State both IUPAC names and special names for substituted naphthalenes and
anthracenes
• Draw structures when given names of substituted naphthalenes and anthracenes

2.1 Nomenclature of monosubstituted benzene


Like other organic compounds benzene and other aromatic compounds have various
method of naming. However, for continuity IUPAC (systematic) naming is preferred and
the following rules will apply.

Rule 1. For monosubstituted compounds the name should read in such a way that the
prefix of the substituents appear before the word benzene.

F Cl Br I NO2

Fluorobenzene Chlorobenzene Bromobenzene Iodobenzene Nitrobenzene

16
Some mono-substituted aromatic compounds have special names that are used instead of
the systematic names.

CH3 OCH3
NH2 OH

Aniline Phenol Toluene Anisole


(Aminobenzene) (Hydroxybenzene) (Methylbenzene) (Methoxybenzene)

COOH SO3H CHO


COCH3

Benzoic acid Benzenesulphonic acid Acetophenone Benzaldehyde


(benzenesulfonic acid)

2.2. Nomenclature of substituted benzene


Rule 2. If there are several substituents on the ring then the relative positions are shown
by numbering the carbon atoms to which they are attached. Numbering starts from the
most electronegative group and the same of the numbers must be the minimum.

NO2
NO2 Note:
o o
Cl 1 - 2 ortho (o)
m m 1 - 3 meta (m)
p Br 1 - 4 para (p)
3-Bromonitrobenzene 2-Chloronitrobenzene
(m-Bromonitrobenzene) (o-Chloronitrobenzene)

Rule 3. When a group with special name is present the parent special name is used.

OCH3 CH3
NH2
OH
HO2C

NO2
NO2 I SO3H
Br

3-Nitroaniline 2-Nitrophenol 4-Bromoanisole 2-Iodobenzoic acid 4-Toluenesulfonic acid


(m-Nitroaniline) (o-Nitrophenol) (p-bromoanisole)

Rule 4. When two or more similar groups are present then the prefix di, tri, tetra, penta
hexa etc is used to denote the number of groups present.

17
Cl
Br NO2
Cl
Cl
Br

O2N Cl
Cl NO2
Cl

1,3-Dichlorobenzene 1,2-Dibromobenzene 1,3,5-Trinitrobenzene 1,2,4,5-tetrachlorobenzene


(m-Dichlorobenzene) (o-Dibromobenzene)

2.3 Nomenclature of other benzene derivatives


Rule 5. If more than two groups that are different are attached to the benzene, the
numbering is done from the most electronegative to give the minimum sum possible and
prefix must be in alphabetical order.

Cl
Br NO2

NO2 Br
Cl Br
I Cl
3-Bromo-1-Chloro-5-Iodobenzene 5-Bromo-3-chloronitrobenzene 2-Bromo-3-chloronitrobenzene

Rule 6. If there is a special group, then the derivative is given the special name and
numbering started from there.
OH NH2
Cl Br Br

O2N NO2
NO2 Br CH3

2-Chloro-4-nitrophenol 2,4,6-tribromoaniline 2,6-Dinitrotoluene

Rule 7. If there is an alkyl group that is not branched, the benzene nucleus is taken as the
parent.

CH3 CH2CH3
CH2CH2CH3

Methylbenzene Ethylbenzene Propylbenzene


(Toluene)

18
Rule 8. It there are two or more alkyl groups the longer one takes preference.

CH2CH3

CH2CH2CH3

3-Ethylpropylbenzene

Rule 9. If the alkyl group is branched, the alkyl group is taken as the parent and benzene
nucleus as a substituent.

2-Methyl-1-phenylpropane 2-Methyl-2-phenylpropane 3-Phenylheptane

Rule 10. If the chain is branched and benzene ring has further substituents on it, then the
branched chain still taken the parent.

NO2
CH3
2-(4'-Methyl-3'-nitrophenyl)-2,5-dimethylhexane

Note:
= C6H5 = Phenyl group = Ph

CH2 = C6H5CH2 = Benzyl group

O
C6H5CO = Benzoyl group = Bz
=

19
2.4 Nomenclature of derivatives of naphthalene
Unlike benzene the carbon atoms of naphthalene are not equivalent. It has three types of
carbon centers as indicated below. The bridging carbon atoms lack the hydrogen atoms
and are not numbered, as they cannot be involved in any form of substitution. As the
carbon atoms are not equivalent in naphthalene, introduction of a substituent gives two
isomers. Before any substituent is introduced both rings are equivalent. For that there are
two alcohols of naphthalene and like phenol, they have special names. If substituents are
more than one, the numbering follows the order shown above. Examples are shown
below.

8 1 α α
7 2 β β
6 3 β β
5 4 α α

Fused carbon atoms

8 1 a a
7 2 b b
6 3 b b
5 4 a a

Fused carbon atoms

OH NH2 NO2
OH NO2

NO2 NO2
1-Naphthol 2-Naphthol
(1-Hydroxynaphthalene) (b-naphthol) 2,4-Dinitro-1-naphthalamine 1,5-Dinitronaphthalene
(a-Naphthol)

OH CH3 Br
SO3H Cl

H2N

6-Amino-2-naphthalene 7-Chloro-1-naphthol 1-Bromo-8-methyl


sulfonic acid naphthalene

2.5 Nomenclature of anthracene


The numbering in anthracene is different from that of benzene and naphthalene in that is
middle ring is numbered last. That is, the two carbon atoms of middle ring are numbered

20
9 and 10. Due to presence of more positions anthracene has very many varied derivatives.
A few selected examples are given below.

8 9 1
7 2

6 3
5 10 4 OH
Anthracen-9-ol
(9-Anthrol)
OH OH

OH
Anthracen-1-ol
Anthracene-9,10-diol
(1-Anthrol)

O O

O
9,10-Anthraquinone 9-Anthrone

OH NO2

9,10-Dihydro-anthracen-9-ol NO2

9,10-Dinitroanthracene

SO3H SO3H

HO3S
Anthracene-2-sulfonic acid Anthracene-2,6-disulfonic acid

2.7 Nomenclature of phenanthrene


Numbering in phenanthrene is similar to that of anthracene. It is important to note the
difference between the two three ringed aromatic class of compounds. In anthracene the
three ring are fused linearly while in phenanthrene the fusion is not linear but in such a

21
way that the two middle carbon atoms be on the same side. In anthracene there are many
positions thus many derivatives.

3
4 2
5
1
6

7 10
OH
8 9

Phenanthrene Phenanthren-9-ol

OH O
OH O
Phenanthrene-9,10-diol Phenanthrene-9,10-dione

O
10-Hydrophenanthren-9-one 9,10-Dihydro-phenanthrene

HO

OH

Phenanthren-1-ol Phenanthren-4-ol

Cl

NO2

NO2
Cl
1,10-Dinitrophenanthrene 2,7-Dichloro-phenanthrene

22
2.8 Summary
In IUPAC nomenclature of benzenes, the name benzene appears as a suffix if the name is
based on benzene.
Relative positions; 1:2, 1:3 and 1:4 on benzene ring are referred to as ortho (o), meta (m)
and para (p) respectively and sometimes used in nomenclature.
Numbering starts at the special group if present on benzene ring and hence special name
is used.
When branched alkyl group is attached to benzene ring, its taken as the parent and
benzene becomes a substituent.
Naphthalene has different positions while anthracene and phenanthrene have three each.
In anthracene and phenanthrene the two carbon atoms of the middle ring is numbered last
as positions 9 and 10.

2.9 Questions and Solutions


Questions

Q1. Give the systematic (IUPAC) names for the following compounds

NH2 Br
Br
COCl CH3
(i) (ii) (iii)
C C
H3C H
Cl NO2

OCH3 OH
CH2CHCH3 O3N NO2
(iv) (v) (vi)

Br CO2H
F NO2
Br SO3H
NO2
OH (viii) Cl
(vii) (ix)
Cl
NO2 OH
OH

Q2. Draw structures of the following compounds.

(i) 4-Cyclohexylanisole
(ii) 3-methylphenylbenzoate
(iii) 3-Bromo-5-nitrobenzene sulphonic acid

23
(iv) 3-Chloro-7-fluoro-1-naphthol
(v) 2-amino-4-butoxybiphenyl
(vi) 2,7-anthracenedisulphonic acid

Solutions
1. (i) 2-Bromo-4-chloroaniline
(ii) 2-Bromo-5-nitrobenzoyl chloride
(iii) (Cis)-3-Phenyl-2-butene or 3-phenylbut-2(Z)-ene
(iv) 3-Bromo-5-methoxybenzoic acid
(v) 1-Phenyl-2-(3’-fluorocyclopentyl)propane
(vi) 2,4,6-Trinitrophenol
(vii) 5-Nitro-2-naphthol
(viii) 8-Bromo-3-chloro-5-hydroxy-1-naphthalene sulphonic acid
(ix) 6-Chloro-9-nitro-1-anthrol

2. Structures

O
C
(i) H3CO (ii) H3C O

SO3H OH
(iii) (iv)
F

O2N Br Cl

HO3S SO3H
O (vi)
(v)

H2N

24
CHAPTER 3

REACTIONS OF BENZENE

3.0. Introduction
In chapter one, we looked at what aromaticity is and chapter two, we looked at how
various benzene derivatives and benzenoids are named using IUPAC names and some
times special names. In this chapter, we will look at influence of aromaticity to reactions
shown by benzene. Detailed method for writing EAS reaction mechanisms will be
discussed. Halogenation, nitration, nitrosation and Friedel-Crafts reaction will be used to
illustrate EAS reactions shown by benzene.

Objectives
By the end of this lesson, you should be able to:

• Explain using mechanism, the reduction of benzene.


• Describe using curly arrows, the general mechanism for electrophilic aromatic
substitution (EAS) reaction.
• Write equations to show the generation of specific nucleophiles for EAS
reactions.
• Write down reaction mechanisms for halogenation, nitration, sulphonation,
nitrosation and Friedel-Crafts reactions.
• Apply EAS patterns to synthesize substituted benzenes.
• Draw structures when given names of disubstituted benzene.

3.1 Reduction of benzene


As had been stated earlier benzene prefers to undergo other reactions where aromaticity
is retained. However, under pressure and in presence of a metal catalyst like nickel,
platinum or palladium, benzene reacts with three moles of hydrogen to form cyclohexane.

H2/Ni
High pressure H2/Ni H2/Ni
+
slow fast fast

Benzene Cyclohexadienes Cyclohexene Cyclohexane

First step is slow due to aromatic character of benzene as compared to cyclohexadienes.


The intermediates in this reaction, cyclohexadienes and cyclohexene, cannot be isolated
because they under go reduction very fast.

25
Birch Reduction

This is a form of limited reduction discovered by Birch, an Australian chemist. The


method is for producing 1,3-cyclohexadiene from benzene.

Na, NH3, t-BuOH


+

1,4-Cyclohexadiene 1,3-Cyclohexadiene
Major product (Minor product)

The reaction involves the treatment of benzene derivative with sodium (or lithium) metal
in liquid ammonia. The metal dissolves the ammonia and so these types of reactions are
also referred to as a dissolving metal reduction.

Mechanism:

- -
Na.
. .

A B

Note that intermediate B is more stable than intermediate A because of electrons


(negative charge and the radical) are furthest from each other hence least repulsion. Each
of these intermediates reacts with further with ammonia and sodium to form their
respective products.

-
. H NH2 H NH2
. -
A
Na. 1,3

- H NH2
. H NH2
. -
B Na. 1,4

Although 1,3 product is stable due to conjugation, 1,4 product is preferred and formed
due to its formation from a stable intermediate.

26
3.2 Electrophilic aromatic substitution (EAS) reactions of benzene:
A general mechanism
These are the most common reactions that benzene and other aromatic compounds
undergo. Due to the π electrons cloud the aromatic ring acts as a nucleophile that is
attacked by an electrophile. Generally the mechanism involves formation of sigma
complexes followed by elimination, which restores aromaticity to the ring. [E+ =
Electrophile; B- = Base]

H H
E E + H
E + E
+ +
Sigma Complexes

H E
B- E
Then,
+

3.3 Halogenation
Bromination: Bromine molecule itself is not sufficiently reactive to participate in EAS.
A Lewis acid catalyst must be added.

Br2 Br
+ HBr + FeBr3
FeBr3

Initial step is formation of a Br2-FeBr3 complex that serves as the E+ in generation


mechanism.

2Fe + 3Br2 2FeBr3


δ+ δ−
FeBr3 + Br-Br Br-Br-FeBr3

H Br Br- FeBr3
δ+ δ− Br
Br-Br-FeBr3 +
+ -
FeBr4
Resonance-stabilized + HBr + FeBr3
Sigma complex

27
Formation of the sigma complex is the rate-determining step. This step is highly
endothermic, while the reaction as a whole is exothermic.

Br2 H Br Br

FeBr3 + Fast
Slow
RDS Resonance-stabilized
Sigma complex

Figure 4. Energy diagram of bromination of benzene

Chlorination: Chlorination has an analogous mechanism to that of bromination. AlCl3


can be used as the Lewis acid catalyst.

Cl2 Cl
+ HCl + AlCl3
AlCl3
Chlorobenzene

Iodination: Requires more specialized conditions. Nitric acid (HNO3) is used as a


promoter (not catalyst). Iodonium ion is the electrophile generated with the help of nitric
acid as shown below.

28
I2 I
+ NO2 + H2O
HNO3
Iodobenzene

2H+ + 2HNO3 + I2 2I+ + 2NO2 + 2H2O

3.4 Nitration
Direct reaction of nitric acid with benzene is slow. The reaction is sped up by the addition
of sulfuric acid.

HNO3 NO2
+ H2O
H2SO4

Nitric acid and sulfuric acid react to give the nitronium ion, which is the active
electrophile (E+).

O O
N+ O O
N+ -
-O OH H O S OH -O OH2 + O S OH
O O

+
O N O + H2O
Nitronium ion

Other nitrating agents are nitronium perchlorate (NO2+ClO4-) and nitronium fluoroborate
(NO2+BF4-).

3.5 Sulphonation
Reaction of benzene with SO3 in H2SO4 (fuming sulfuric acid) gives benzene sulphonic
(sulphonic) acid.

SO3
SO3H
H2SO4
Benzene sulfonic acid

29
SO3 is a strong electrophile and unlike nitration and halogenation, sulphonation is
reversible.

-
O O
S S+
O O O O

H2SO4
SO3H + SO3
Heat

3.6 Friedel-Craft alkylation


Reaction of aromatic compounds with alkyl halides in presence of Lewis acid, affords
alkylated products.

AlX3 R
+ RX + HX

X = Cl, Br, I R = Alkyl group

AlCl3
+ Cl + HCl

The first step the reaction is generation of the carbocation, which is the electrophile. The
next step is attack of the electrophile to the aromatic system to give the sigma complexes,
followed by elimination.

Cl AlCl3 -
+ + AlCl4

H -
AlCl4
+

Resonance stabilized
Sigma complexes

For 20 and 30 alkyl halides, the ‘naked’ carbocation is likely involved, as they are
relatively stable. The carbocations formed from 10 alkyl halides are much less stable and
so a complex between the alkyl halide and Lewis acid is probably the species attacked by
the aromatic ring.

30
H CH2CH3 -
AlCl4 CH2CH3
H2C Cl AlCl3
Ch3 +

Note: There are three limitations to alkylation reactions.


1). Only works well for benzene, halobenzenes and activated ring systems.
2). Electrophilic species are carbocations that are prone to rearrangement.
3). Multiple alkylations frequently occur because the alkylated product is more
activated then benzene itself.

H
H CH3
C
C CH3 Not Formed
AlCl3 CH3
+ +
CH3 H Product formed via
H CH3 0
C C CH3 rearrangement of a 2 carbocation
H3C C C Cl 0
CH3 H to a 3 carbocation
CH3 H

H CH3
AlCl3 H H
H3C C C Cl + Rearrangement +
H3C C C CH3 H3C C C CH3
CH3 H CH3 H CH3 H

20 carbocation 30 carbocation

3.7 Friedel-Crafts acylation


An acyl group has a carbonyl group attached to hydrogen or an alkyl group. The reaction
is of acyl halide and a Lewis acid.

O O
AlCl3 +
+ HCl
R
R Cl

31
Lewis acid assists in generation of an ‘acylium ion’ as the electrophile

O O +
O -
AlCl3 + + AlCl4
R Cl R
R
Acylium ion

Unlike the Friedel-Crafts alkylation the acylation reaction does not suffer from
rearrangement of the electrophile nor is the product susceptible to further reaction. The
acylation reaction can be used to synthesize alkyl benzenes indirectly. For example,
Clemmensen reduction can be used where carbonyl group to CH2 groups upon reaction
with a zinc/mercury amalgam.

CH3 Zn (Hg) CH3

HCl

3.8. Nitrosation
The reaction involving nitrous acid is known as nitrosation. The electrophile involved is
nitroso cation generated by nitrous acid.

NO
+ HNO2 + H2O

Nitrosobenzene
HONO OH -
NO+
+

H
NO OH- NO
+ NO +
+

32
3.9 Summary

Benzene undergoes mainly EAS reactions to give various substituted products due its
aromaticity.
Products of EAS of benzene are used as either intermediates or final products or as
precursors on industrial scale.
Fiedel-Crafts alkylation gives mixed products due to rearrangement of the resulting
carbocation. Therefore, it is not a good method to synthesis alkylated benzene
derivatives.
Fiedel-Crafts acylation does not have rearranged products thus preferred.

3.10 Questions and solutions


Questions

Q1. (a) By use of reaction mechanisms show how the following transformations are
carried:
(i) Benzene to nitrobenzene
(ii) Benzene to ethylbenzene
(iii) Benzene to nitrosobenzene
(iv) Benzene to benzene sulphonic acid
(v) Benzene to Iodobenzene

Q2. Explain the following by use of appropriate structures and/or equations.


(i) Friedel-Craft alkylation reactions leads in formation of a mixture alkylated
benzene compounds.
(ii) In nitration of benzene concentrated sulphuric acid is required.

Solutions

1. Refer to the notes within this chapter.

2.(i) The carbocation formed when the alkyl halide react with a Lewis acid is prone to
rearrangement leading to more alkylated products. Consider 1-chloropropane.

AlCl3
CH3CH2CH2Cl CH3CH2CH2+ + AlCl4-

Rearrangement

+
CH3CHCH3 a more stable
carbocation

33
These two carbocations will react with benzene to give two different products
although the secondary carbocation will form the major product, isopropylbenzene.
That was not the initially expected product.
H
CH2CH2CH3 H3C C CH3

Propylbenzene
2-Phenylpropane
Minor Product
(Isoprylbenzene)
Major Product

It is also noted that the alkylated product formed is activated to electrophilic


substitution reactions than benzene thus forms multi-alkylated benzene products at
ortho and para positions.

R R
R R R R R

R R
R R

34
CHAPTER 4

REACTIONS OF BENZENE DERIVATIVES

4.0 Introduction
In chapter three we looked at EAS reactions of benzene with various nucleophilic
reagents. In this chapter we are going to study the effect of groups attached to benzene
ring in terms of how they affect rate of electrophilic aromatic substitution (EAS)
reactions and position at which the incoming substituent gets attached to the ring. These
reactions will also be used to show the synthesis of various substituted benzenes.
Industrially useful substituted benzenes will also be discussed.

Objectives

By the end of this lesson, you should be able to:

• Explain the effect of electron donating group on benzene on the rate of EAS and
orientation of an in coming nucleophile
• Explain the effect of electron withdrawing group on benzene on the rate of EAS
and orientation of in coming nucleophile
• Describe how the rate of EAS reaction of benzene and substituted benzene is
determined
• State characteristics of common ortho-para and meta directors
• Explain using resonance structures, the effect of a group on benzene that donates
electrons by induction and that releases electrons by resonance on EAS
• Explain why halogens are deactivators yet they are ortho-para directors to an
incoming electrophile
• Explain the effect of two substituents on benzene to the rate reaction and
orientation of incoming substituent
• State some of industrially useful substituted benzenes

4.1 Effect of substituents on EAS patterns and orientation

4.1.1 Nitration of Toluene


Like benzene, toluene undergoes electrophilic aromatic substitution reactions. The
reaction is faster than that of benzene. Nitration reaction, for example, is about 25 fold
faster. The reaction gives three products, two major and one minor and almost not
realized.

35
CH3
CH3 HNO3 CH3 CH3
+
H2SO4 +
NO2 O2N
NO2

o-Nitrotoluene m-Nitrotoluene p-Nitrotoluene


40% 3% 57%

If nitration were random, a 20:40:40 para:meta:ortho ratio would be produced. This is


due to the fact that there are two ortho positions, two meta positions and one para
position. However the situation is not observed experimentally. The substituents have
effects on EAS of benzene derivatives. In the above case of nitration of toluene the ortho
and para products are favored.

From the above observations on nitration of toluene two conclusions can be made about
the methyl group attached to benzene ring:
1. It is an activating group because the reaction is faster than that of
benzene towards electrophilic substitution.
2. It is an ortho–para director.

Recall that the rate-determining step is the formation of the sigma complexes. The
complexes leading to the ortho and para products are more stable than that leading to the
meta product. Accordingly, the transition states leading to the ortho and para products
are of lower energy than those leading to the meta product.

CH3 CH3
+ + CH3
Ortho
NO2 NO2
H NO2
H + H
Two with 20 carbocations, one with a 30 carbocation.
CH3
HNO3 CH3 +
CH3 CH3
H2SO4 Meta + +
H NO2 H NO2 H NO2
All three with 20 carbocations

Para CH3 CH3


CH3 +
+
H H
H
O2N +
O2N O2N

Two with 20 carbocations, one with a 30 carbocation.

36
Figure 5. Energy diagram of nitration of toluene

4.1.2 Nitration of nitrobenzene


Unlike the case of nitration of toluene, nitration of nitrobenzene is a very slow reaction. It
is about 105 times less reactive to EAS reactions than benzene. The reaction takes place at
higher temperature when compared with the nitration of benzene. The reaction leads to
formation of three products, with m-dinitrobenzene, the meta product predominates and
para product almost not realized.
NO2
NO2 HNO3 NO2 NO2
+
H2SO4 1000C +
NO2 O2N
NO2

o-Dinitrobenzene m-Dinitrobenzene p-Dinitrobenzene


6% 90% 0.7%

The nitro group is electron withdrawing which deactivates the ring towards EAS. The
group removes electron density from the ring and slows down the reaction.

O O-
N N
+ O- + O

From the above observations on nitration of nitrobenzene two conclusions can be drawn
about the a nitro group attached to benzene ring:
1. It is a deactivating group because the reaction is slower than that of
benzene towards electrophilic substitution.
2. It is a meta director

37
The sigma complex leading to the meta product is less destabilized than those leading to
the ortho and para products.

O
+N -
O NO2 NO2
+ +
Ortho
NO2 NO2
H NO2
H + H
Especially destabilized
NO2 (Adjacent + charges)

HNO3 NO2 +
NO2 NO2
H2SO4 Meta + +
H NO2 H NO2 H NO2

O
NO2 -
Para N O NO2
+ +
+
H H
H
O2N +
O2N O2N

Especially destabilized
(Adjacent + charges)

Generally groups that deactivate the ring are usually meta directors. Other deactivating
meta directing groups include, ketones, esters, nitriles sulphonic acids and ammonium
salts. All are either positively charged, or have a resonance form in which a positive
charge is immediately adjacent to the ring.

4.1.3 Determination of relative reactivity of reaction


1. Time required for a reaction to occur under identical conditions
2. The severity of conditions (temperature, pressure and concentration) required for a
comparable reaction to occur under other identical conditions. For example,
nitration of benzene and nitrobenzene for one hour is at 600C and 900C,
respectively. Benzene is therefore more reactive than nitrobenzene.
3. Competitive reactions whereby a mixture of equal moles of compounds is be made
to compete for a limited amount of reagent and quantitative analysis of the product
and the reactants done.

4.1.4. Orientation of the nitro group


The table below presents some information on the nitration of monosubstituted benzene.
From the table one can be able to come up with a classification of the groups as ortho-
para directors or meta directors.

38
Relative percentage of product
Group attached Ortho Meta Para
to benzene
-OH 55 0 45
-CH3 40 3 57
-OCH3 45 ≈1 55
-NH2 50 0 50
-CH2CH3 35 ≈1 63
-Cl 35 1 64
-Br 40 0 60
-NHCOCH3 19 2 79
-NO2 6 90 <1
-CO2H 20 80 0
-CN - 81 -
-SO3H 21 72 7
-CHO - 72 -
-+N(CH3)3 0 89 11

Strong activating agents (ortho-para directing): -NH2, -NHR, -NR2, -OH, OMe
Moderately activating (ortho-para directing): -NHCOCH3
Weakly activating (ortho-para directing): -C6H5, -CH3, -CH2CH3

Deactivating agents (ortho-para directing): Halogens (-F, -Cl, -Br, I)


Deactivating agents (meta directing): -NO2, -CO2H, -CN, -SO3H, -CHO etc

Note:
a) An activating group activates all the positions on the benzene ring including the
meta position but activates ortho and para positions more than meta.
b) A deactivating group deactivates all the positions on the benzene ring including
the meta positions but less than the ortho and para.

4.2 Groups that donate electrons to the benzene


There are two types of groups under this category. There are those groups that donate
electrons by inductive effect and those that donate by resonance.

Groups that donate electrons by inductive effect are groups that do not have lone pair of
electrons to donate for resonance stabilization. But they are electron releasing by
inductive effect. Thus they increase the electron density to the benzene ring and hence the
ring becomes more attractive towards an incoming electrophile than benzene itself. They
are however termed weak activators. Alky group, methyl, ethyl, propyl etc, represent this
class (refer to nitration of toluene).

39
Groups that release electrons by resonance have lone pairs of electrons, and are powerful
activators of ortho and para positions. They include amino (-NH2), and hydroxyl (-OH)
groups. They release electrons by means of resonance. Consider a case of nitration of
aniline.

+
NH2

NO2
.. H
NH2 NH2
+ NH2
+
Ortho
NO2 NO2
H NO2
H + H

NH2
HNO3 NH2 +
NH2 NH2
H2SO4 Meta + +
H NO2 H NO2 H NO2

NH2 ..
Para NH2 NH2
+
+
H H
H
O2N +
O2N O2N

+
NH2

H
O2N

The lone pair of electrons of nitrogen provides an additional stabilization of the sigma
complex by resonance donation of the electrons. There is an additional resonance form
for the complexes leading to the ortho and para products. Hence these sigma complexes
are more stable.

Under some conditions aniline is so reactive that it undergoes EAS without a catalyst.
Bromination occurs without the Lewis acid and substitution can takes place in three
positions.

NH2 NH2
3Br2 Br Br
H2O, NaHCO3

Br

40
Sodium bicarbonate is required to react with the HBr that forms. Otherwise, the basic
amino group protonates and EAS slow down. Protonated amino group is an electron-
withdrawing group and removes electrons density from the ring and slows down the
reaction.

NH2 + -
NH3 Br
HBr

4.3 Groups that withdraw electrons from the benzene ring by induction
and release electrons by resonance
They display two opposing effects:
1. They are electronegative and hence deactivate the ring through
inductive effects.
2. They can donate electron density through resonance and hence activate
the ring through resonance through formation of a ‘halonium ion’
(C=X+).

Net effect is that the rate of EAS is slower for halobenzene than benzene, but these
groups give ortho and para products, as the major ones.

Br Br Br
NO2
HNO3/H2SO4
+ + O2N Br
NO2
ortho ~35% meta ~1% para ~64%

It is observed that like the sigma complexes of aniline, chlorine atom can contribute a
lone pair of electrons and the sigma complexes for ortho and para increased by one each
through formation of the ‘halonium ion’.

41
+
Cl

NO2
.. H
.. :
Cl Cl Cl
+ +
Ortho
NO2 NO2
H NO2
H + H

Cl
HNO3 Cl +
Cl Cl
H2SO4 Meta + +
H NO2 H NO2 H NO2

Cl ..
Para .. :
Cl Cl
+
+
H H
H
O2N +
O2N O2N

+
Cl

H
O2N

4.4 Effect of a third substituent on the benzene ring


Presence of already two substituents on the benzene ring complicate the rules mentioned
earlier if the third substituent is to be introduced. However, a few general rules for some
cases can be predicted by combining the effects of each of the two groups.

If the two substituents direct an incoming group to the same position(s), then that will be
the principal position of the third substituent. That is, the two substituents may be located
in such a way that their directive influence reinforces each other. A few examples are
cited below.

42
SO3H
NHCOCH3 CH3
H3C NO2
NO2 CN
Hindered
CH3
Highly deactivated
Predicted site if EAS
works at all

Reaction between two substituents generally not favored due to steric hindrance.
Although the two methyl groups in m-xylene would reinforce each other no reaction
takes place for the ortho position between the two groups.

If two deactivating groups are present, regardless of their position, it may be difficult to
effect a third substitution (case of m-nitrobenzene sulphonic acid above).

If two groups conflict in their directive effects, the more powerful activator will exert the
predominant effect (NH2>OH>OCH3>NHCOCH3>C6H5>CH3>halogens>meta directors.

X
X2
HO Cl HO Cl

H3C NHCOCH3 Major

Br2/FeBr3 Br
H3C NHCOCH3

H3C NHCOCH3 Minor


Br

4.5 Summary
Rate of EAS reaction of monosubstituted benzene with an incoming nucleophile and the
orientation is affected by nature of the substituent already on the ring.
Electron withdrawing groups (activators) are meta directors.
Electron donating groups (deactivators) are ortho and para directors.
Halogens are ortho and para director although they are deactivators.
The position of the third incoming substituent on benzene ring does not follow the usual
EAS patterns.
Products of EAS of benzene have wide range of industrial uses.

43
4.6 Questions and solutions
Questions

Q1. Explain the following by use of reaction mechanisms and resonance structures.

(a) A keto group attached to benzene ring is deactivating and directs the incoming
electrophile to the meta.
(b) Birch reduction of benzoic acid (A) gives compound (B) and not compound
(C).

CO2H CO2H CO2H


Li, NH3 (l)

EtOH
A B C

Q2. Suggest the major organic compounds (D-H) in the following reactions.

NHCOCH3
OH
(i) Br2, CH3CO2H (ii) HNO3
D E
H3C
NH2
NO2
(iii)
Cl2/H2O (iv)
F HNO3/H2SO4
G
NO2 CH3
O
OCH3 AlCl3
(v) + O H

Solutions

1. (a) The carbon of the carbonyl group is polarized in such a way that the carbon atom of
the carbonyl carries a partial positive charge while the oxygen carries the partial
negative charge. That is, carbon is in need of electrons. These electrons are supplied
by the benzene ring. As the electron density of the benzene is reduced, the carbonyl
group is therefore termed as a deactivating group because the benzene ring
reactivity towards electrophilic attack has been lowered.

δ−
O
+

R R

44
Any electrophilic attack, which would lead to resonance structures whereby the benzene
carbon attached to the carbonyl group carbon carries a positive charge, is unfavorable.
This is noted in the ortho and para electrophilic attacks. Although the whole ring is
deactivated, ortho and para position are more deactivated than the meta where the
positive charge does not fall on the carbon atom of the benzene attached directly to the
carbonyl carbon as observed in the resonance structures below.

COCH3 COCH3 COCH3


COCH3
+ +
HNO3/H2SO4
H H H
meta attack NO2 NO2
+ NO2

COCH3 COCH3 COCH3 COCH3


HNO3/H2SO4 + NO2 NO2 NO2
H H H
ortho attack + +

COCH3 COCH3 COCH3 COCH3


HNO3/H2SO4
+
para attack +
+
H NO2 H NO2 H NO2

1 (b). A carboxylic group is an electron-withdrawing group as the carboxyl carbon highly


deprived electrons by the two oxygens attached. The mechanism below will help to
explain why the product is formed.

CO2H HO C O HO O-

EtOH
Li -
+ etc
. .
Negative charge resonance stabilized
by the electron deficient CO2H group

HO C O HO O CO2H
Li, EtOH
.
+ etc
.
Favored product
Lower energy pathway

45
CO2H CO2H CO2H CO2H CO2H
- - EtOH Li
Li -
. . . EtOH

Negative charge not stabilized by the


CO2H group. (Higher energy pathway)
-Not fovorable

2.

NHCOCH3 NH2
OH Cl
(i) D = (ii) E = (iii) F =
H3C NO2
Br NO2
NO2
NO2 OCH3
(iv) G = (v) H=

O C
CH3 CH2CH2CO2H

46
CHAPTER 5

ARENES AND ARYL HALIDES

5.0 Introduction
In chapter four, we looked at EAS reactions of benzene and its derivatives and effect of
substituents on rates of EAS reactions as well as their orientation patterns. In this chapter,
preparation of arenes and reactions shown by arenes will be studied. Discussion of EAS
reaction of aryl halides will be done in addition to those reactions that will lead to
removal of the halogen from the ring. Reactions involving aryl halides useful in the
synthesis of important compounds will also be mentioned

Objectives

By the end of this lesson, you should be able to:

• Describe using equations preparation of some common arenes.


• Explain with the help of mechanisms why different products are obtained when
arenes are halogenated in dark and in presence of light.
• Predict reaction products when unsaturated arenes undergo halogenation in the
presence of Lewis acids.
• Write down oxidation and reduction products of styrene.
• Explain deactivating effect of halogens using resonance structures.
• Describe EAS reactions of aryl halides.
• Describe how a halogen substituent is removed from activated and inactivated
aryl halide.
• State some of the important arenes, aryl halides and compounds obtained from
them.

5.1 Arenes
Arenes are aliphatic aromatic hydrocarbons. Recall the methods of naming arenes
(chapter 2) and methods of preparations. Note that if a double is present, depending on
the position, geometrical isomerism may be exhibited.

H H H3C
CH2 CH3 H
H H
H H
3-Phenyl-1-propene (Trans)-1-Phenylpropene (Cis)-1-Phenylpropene

47
Arenes are of low polarity and insoluble in water but soluble in organic solvents like
CCl4, ether, hexane etc. Disubstituted para isomers have higher melting point due to
better packing.

5.2 Reactions of Arenes


Arenes undergo most of the reactions of benzene ring, that is, EAS reactions. They can
also undergo reactions involving the side chain. Recall that they are they are ortho-para
directors and activates the ring by inductive effect. The para product usually
predominates over the ortho product due to steric effects.

Note that the halogenation with Lewis acid is carried out in darkness. In presence of light
the side chain will react with the halogen.

CH3
CH2CH3
Br2/Light H
Br

CH2CH3 CH2CH3 CH2CH3


Br2/FeCl3
+
Dark Br Br
Minor Major

5.2.1. Halogenation of Arenes


Halogenation of the side chain is a radical mediated reaction. 1-Bromo-1-phenylethane,
shown above is the only product due to stability of the intermediate radical involved
being stabilized by the phenyl group by resonance. This is a reactivity selectivity
principle.

hv . .
Br2 Br + Br
.
CH2CH3 CHCH3
. HBr
+ Br +

CH3
.
C H .
CHCH3 + Br2 + Br
Br

Only product

Chlorination of ethylbenzene, however, gives two products with 1-chloro-1-phenylethane


(91%) predominating over 1-chloro-2-phenylethane (9%).

48
Question: What are the terminating products of the halogenation of ethylbenzene in
presence of light?

Unsaturated arenes undergo halogenation in absence of the Lewis acid just like alkenes.

Br2 H
Br 1,2-Dibromo-1-phenylethane
Br H
H

Br2/FeCl3
+ Bromostyrenes
Br Br
major

HBr H
CH3 1-Bromo-1-phenylethane
Br (Markovnikov's product)
Styrene

H
HBr 1-Bromo-2-phenylethane
CH2Br (Anti-Markovnikov's product)
PhCOOOCOPh H
(Dibenzoylperoxide)

5.2.2 Reduction reactions of arenes


The reaction of styrene can occur both on the side chain and also on the ring. Consider
reduction with hydrogen in presence of nickel.

H2/Ni CH2CH3

200C, 2-3 Atm Ethylbenzene

CH2CH3
Styrene H2/Ni
Ethylcyclohexane
1150C, 110 Atm

In presence of hydrogen peroxide in acetic acid, styrene is converted to a glycol, which is


oxidized further by oxidative cleavage to benzoic acid. Any side chain with the carbon
atom that is directly attached to the ring (benzylic carbon), and has at least two
hydrogens, is oxidatively cleaved by strong oxidizing agent to benzoic acid.

H OH
H2O2/HCO2H CO2H
CH2OH KMnO4

Styrene
Phenyl-1,2-ethanediol Benzoic acid
(a Glycol)
49
CO2H
-
KMnO4, OH, Heat
H+ CO2H
CO2H
-
KMnO4, OH, Heat
No reaction
H+

Question: How are (i) saturated and (ii) unsaturated arenes prepared?

For the saturated arenes refer to Friedel-Crafts alkylation. Recall that the method has its
limitations. It was also mentioned that the limitations could be avoided by use of Friedel-
Crafts acylation later reducing the acylated product by Clemmensen reduction (HCl in
presence of Zn/Hg) or Wolf-Krisher reduction (N2H4 in presence of a base). Unsaturated
arenes are prepared by normal reduction of acylated products.

Note that if the alkyl halides have more than one halogen atom, more than one phenyl
groups will be present in the products.

AlCl3
2 + CH2Cl2 Diphenylmethane

AlCl3
3 + CHCl3
H Triphenylmethane

AlCl3
3 + CHCl4
Cl Chlorotriphenylmethane

Triphenylmethane is acidic due to resonance stabilization of carboanion. Use of carbon


tetrachloride does not lead to formation of tetraphenylmethane due to steric factors.

5.3 Aryl halides and nucleophilic substitution reactions


Aryl halides, as had been mentioned earlier, are compounds in which halogen atom is
directly attached to the aromatic ring. Their preparation methods were mentioned in
chapter 3. Aryl halides behave differently from alkyl halides. They do not undergo
nucleophilic substitution that easily as do the alkyl halides. They do not five a positive

50
silver nitrate test as alkyl halides do. In some case they behave like vinyl halides (a
halogen attached to a carbon atom with a double bond), which do not react with
nucleophiles.

X CH2X

or ArX

Aryl halide Not an aryl halide

SN1 or SN2 -
R-X + Nu - R-Nu + X

ArX - No reaction
+ Nu

This behavior of aryl halide is due to the contribution of one of the lone pairs of the
halogen towards extension of delocalisation.

.. .. ..
:X : ..
:X + :X + :X +

- -

Thus the ring as a whole is deactivated towards nucleophilic attack. However, in presence
of an electron withdrawing group at ortho or para (or both) with respect to the halide
activate such a reaction. This is an addition-elimination reaction. Presence of the
deactivating group at meta has no such effect on the aryl halide.

Cl O- Cl
Nu Nu
Cl -
N+ + Nu O- NO2
O N+
O-
N+ O-
ortho O-
Cl Cl Nu
Cl Nu
+ Nu -

para
O N -O N
+ O- -O N NO2
+ O- + O-

The more the electron withdrawing groups there are at ortho and para the easier it is to
replace the halide. Only effected at ortho and para positions. Ring activators cannot
effect the change.

The order of reactivity of aryl halides to nucleophilic substitution is:

51
I > Br > Cl >F

Cl OH
NO2 NO2
NaOH, 1300C

Reflux
OH
Cl
aq NaHCO3 NO2
NO2
1000C
NO2
NO2
OH
Cl
O2N NO2
O2N NO2 aq NaHCO3
350C
NO2
NO2

A variety of products can be prepared depending on the type of substituted aryl halide
and the base used.

Br OCH3
CH3ONa

O2N 1000C O2N


Cl
NHCH3
NO2 NH2CH3 NO2
1600C

I NHC6H5
C6H5NH2

NC CN EtOH, 950C NC CN

Other important reactions involving aryl halides are shown below.

52
Cl CN CO2H
NO2 NO2 NO2
KCN aq H+

CO2R CO2R CO2R

F Li

H2O
+

Li
Biphenyl

OCH3 NaNH2/NH3
OCH3
Br H2N
2-Amino-4-methoxybiphenyl
(4-Methoxy-2-phenylaniline)

Br
THF or Et2O MgBr
+ Mg(s)
Dry
Grignard Reagent

5.4 Removal of halide substituent from inactivated aryl halide


For inactivated aryl halides, the substitution is carried out at high temperatures in
presence of a strong base. The nucleophilic substitution occurs but under a different
mechanism. In the mechanism a benzyne (a benzene a triple bond) is implicated (See
example and the mechanism in the page that follows).
This reaction cannot take place if there is no ortho hydrogen to be lost in the process of
formation of the extra bond.
Cl ONa OH
H2O H+
+ NaOH
High pressure

Dow Process

Br NH3 NH2
+ KNH2 + KBr
-330C

Cl NH2
NaNH2/NH3
+
H3C H3C H3C NH2
50% 50%

53
Mechanism:

Br
H - H NH2
NH2 - NH2
-
NH2 -
NH2 + NH2
CH3 H3C
CH3 CH3

Benzyne Addition of NH2 to the other end of triple bond provides the
intermediate other regioisomer

Cl
H3CO CH3 NaNH2/NH3
No reaction

CF3 CF3
Cl NaNH2/NH3
NH2

The last transformation can only be explained by considering a benzyne intermediate.


The benzyne generated as the intermediate can be trapped in a Diels Alder reaction.

+ O O

Benzyne Furan Diel Alder Adduct

5.5 Summary
Arenes give different products depending on whether is done in presence or absence of
light.
Unsaturated arenas undergo halogenation in absence of Lewis acids just like alkenes.
Arenes are prepared by Frediel-Crafts acylation of benzene followed by reduction.

Halogen substituent on benzene ring deactivates it towards nucleophiles but it is ortho-


para director.
Electron withdrawing groups at ortho and para position on aryl halide makes it easier to
remove the halogen from the ring.
For inactivated aryl halide, the halogen can be removed via benzyne intermediate at high
temperatures.
A halogen on the ring is a deactivator but ortho-para director.
Aryl halides are very important precursor for synthesizing many benzene derivatives.

54
5.6 Questions and solutions

Questions

Q1. With the aid of reaction mechanisms, explain the following observation.

Reaction of ethylbenzene (4) with bromine in presence of light gives compound 5 as the
major product and compound 6 as the minor product.

CH2CH3 CHBrCH3 CH2CH2Br


Br2/ light
+

4 5 6

Q2. Given the following reaction,

CF3 CF3
Cl NaNH2/ NH3

NH2

(i) Write a reaction mechanism leading to the above product.


(ii) Why is the ortho substituted product not formed?

Q3. What is the product in the following transformation?

Br
NO2
+-
Na OCH3

NO2

Solutions

1. See page 53.

2. (a) The reaction involves a benzyne intermediate. Attack of the benzyne by amide ion
would lead to formation of two negatively charged benzenes (see below). The
compound formed is governed by the stability of one of these carboanions. Note
that CF3 group is an electron-withdrawing group, and thus the closer the negative
charge to it the better-stabilized carbocation. That inductive withdrawal of
electrons (not resonance) by the CF3 group more favored if the negative charge is
nearer. This leads to the most favored product (low energy pathway)

55
CF3 CF3 CF3 CF3 CF3
Cl NH2- -
Cl Cl NH2
NH2- -
- +
-
NH2
Benzyne A B
intermediate
NH3

CF3 CF3
NH2

NH2

(b) The ortho product is not formed because the negative charge of the intermediate
(B) is far away from the CF3 group thus less stabilized. Therefore less favored
product due to high energy pathway.

3. The reaction is a nucleophilic substitution reaction, just like the reaction in 2


above. The nucleophile is OCH3 ion, which replaces the chlorine atom, thus lost
as a chloride. The product is 2,4-dinitroanisole.

OCH3
NO2

NO2

56
CHAPTER 6

PHENOLS

6.0 Introduction
In chapter five, we looked at arenes and aryl halides, their reactions and importance as
precursors for variety of industrially useful organic compounds. In this chapter, phenols,
that form the most important derivatives of benzene, will be studied. Physical and
chemical properties of phenols that make then unique will be dealt with in details.
Illustration of synthesis of compounds utilizing phenols will also be studied.

Objectives

By the end of this lesson, you should be able to:

• Explain the physical and chemical properties of phenols.


• Describe laboratory preparation and industrial manufacture of phenol.
• Write resonance structures to illustrate ortho-para directing effect of hydroxy-
substituent on benzene.
• Predict reaction under different conditions products of phenols
• State some of the uses of phenols and their derivatives.

6.1 Physical properties of phenols


Phenols are aromatic alcohols in which hydroxyl group is directly attached to the
aromatic nucleus. Phenols resemble aliphatic alcohols in many ways. However, there
some case whereby there are distinctive differences between the two hydroxyl containing
compounds.

Simple phenols are liquids with high boiling points attributed to powerful hydrogen
bonding. Normally colorless unless oxidized to quinines, which is responsible for the
color of phenol when impure.
H O- O-
O O
N+ N O
N+ O +
O-
HO H O
o-Nitrophenol p-Nitrophenol

Ortho-Nitrophenol has intramolecular hydrogen bonding which lowers its solubility in


water when compared with para-nitrophenol. Generally, the ortho-nitrophenol has lower
boiling point and melting point than meta and para isomers as the intramolecular
hydrogen bonding reduces the intermolecular ones. For this, o-nitrophenol can be steam
distilled while meta and para isomers cannot be steam distilled easily.

57
Phenols are weakly acidic due to resonance stabilization of the phenoxide ion. This
makes them stronger acids when compared with the aliphatic alcohols. Therefore phenol
reacts with strong bases like sodium hydroxide but not with weak ones like bicarbonates.
Alcohols do react with neither weak nor strong bases. This property can be used to
identify and separate phenols from acids and alcohols.

OH O- O O O
-
B
- -

Presence of electron withdrawing groups at ortho and para positions (but not meta)
increase the acidity of phenol. The groups stabilize the resulting phenoxide ion by sharing
some of the negative charge by resonance. The more the groups there are in a phenol
molecule the stronger the acid.

O- O O O-
N+ N+
O- O-

O- O

-
O N
+ O -
O N
+ O-

Phenol salts are soluble in water. Due to the resonance stabilization of the negative
charge on the phenoxide ion it is a poor nucleophile, but a strong base.

Since phenolic moiety is common to all structures, the effect of electron delocalisation is
the same. Thus, the other factor in operation is either ability to donate electrons to the
ring or withdraw electrons from the ring.

58
The following substituted phenols are used to explain the effect:

OH OH OH

OCH3 CH3

pH 9.95 10.30 10.19

OH OH OH

Cl CHO NO2

pH 9.38 7.66 7.14

From the above phenols, electron-donating groups (OCH3, CH3 etc) reduces the acidity of
phenols. This can be explained in terms of how hard it is to break the O-H bond so that
H+ is released. Since these groups increase the electron density of oxygen, it is hard for
the O-H bond to break or ionize. On the other hand, electron withdrawing groups weaken
the O-H bond by reducing the electron density of oxygen and hence easy to break or
ionize. The same effect of substitution on Pka values are also seen in substituted benzoic
acids and substituted protonated anilines.

Note: There is a relationship between the Pka values and reactivity of benzene
ring towards EAS; the more strongly deactivating the substituent is the lower the
Pka values.

6.2 Industrial manufacture of phenol


There are two main processes that are used in industries in manufacture of phenol
normally for industrial use. Dow process, already mentioned earlier, involves alkyl
halide, chlorobenzene heated at 3500C and high pressures in presence of sodium
hydroxide. It is first converted to sodium phenoxide, which hydrolysis in presence of an
acid gives phenol.

Cl ONa OH
3500C, H2O H+
+ NaOH
High pressure

59
The second methods involve cumene (isopropylbenzene) as the raw material. Cumene is
air oxidized to form the corresponding organic peroxide, which on hydrolysis in presence
of an acid lead to formation of phenol.

O2 CH3 O
H2O/H+ + H3C
H3C C
Air oxidation
OOH CH3
CH(CH3)2 OH
Peroxide Phenol Acetone
Cumene

6.3 Laboratory preparation of phenol


Various methods can be used to prepare phenols in the lab.

1. Hydrolysis of diazonium salts

N2+HSO4 OH

H2O N2
+ H2SO4 +

Benzene diazonium
bisulphate

Question: How can you prepare phenol from benzene, through nitrobenzene?

2. Alkali fusion of sulphonates

SO3H OH
NaOH, H2O
3000C

Benzene sulphonic
acid

3. Oxidation of aryl thallium compounds (high yield)

Tl(O2CCF3)2
OCCF3
(CF3COO)2Tl Pb(OAc)4
O
Ph3P

H2O
OH-
OH
H+ O-

60
4. Basic hydrolysis of aryl halides (works only if there are electron-withdrawing groups
attached at ortho or para with respect to the halide. (Refer to the reactions of aryl
halides).

6.3 Reactions of phenols

Acidity:
As mentioned earlier phenols are acidic in nature and will react with strong bases like
sodium hydroxide to form phenoxide salt and water. They partially dissociate in water.

OH O-Na+
+ NaOH
+ H2O

Sodium phenoxide

OH O-
+ H2O + H3O+

6.3.1 Ether and ester formation


Phenols and their salt form ethers when reacted with alkyl halides (William’s synthesis).

ArOH + CH3CH2I ArOCH2CH3

ArO-Na+ + RX ArOR + NaX

H3C OH + BrH2C NO2 H3C OCH2

NO2

Phenols like aliphatic alcohols react with carboxylic acids and their derivatives like acyl
halides and acid anhydrides to form esters. The reaction with the acid is highly reversible
and thus not preferred in ester formation.

RCOCl OCOR

or
OH
(RCO)2O

OSO2R
RSO2Cl

61
6.3.2 Nitration and sulphonation of phenol
Phenol will react with nitric acid to form o-nitrophenol and p-nitrophenol. Sulphuric acid
is not needed in this reaction as the hydroxyl group highly activates the ring for EAS
reactions. The ortho isomer is form in a slightly larger proportion molecules are
stabilized by intramolecular hydrogen bonding.

OH HNO3 OH OH
+
NO2 O2N

Phenol reacts with sulphuric acid to form two products depending on the temperature
under which the reaction is carried out. At low temperatures o-hydroxybenzene sulphonic
acid is form. This product is termed as a kinetic product. At higher temperatures, on the
other hand, p- hydroxybenzene sulphonic acid is the major product. This product is
termed as a thermodynamic product.
OH
0 SO3H
15-20 C
Kinetic product
OH
H2SO4 o-Hydroxybenzene sulphonic acid

OH
0
100 C
Thermodynamic product

SO3H
p-Hydroxybenzene sulphonic acid

6.3.3 Bromination and acylation of phenol


Benzene ring is highly activated by the hydroxyl group such that bromination reaction
takes place in absence of the Lewis acid catalyst. Three positions of the ring are
substituted when bromine is introduced in presence of water. However, mono
bromination at para position when bromine is accompanied by sulphur carbide (CS2) at
00C.

62
OH OH
Br2, H2O Br Br

2,4,6-Tribromophenol
Br
OH
Br2, CS2 OH
p-Bromophenol

Br

Phenol and its derivative can undergo acylation in presence of acid anhydride. In
presence of a Lewis acid the acylated product undergoes a rearrangement where the acyl
group migrates to the ortho or para position depending on the reaction temperature. At
room temperature (250C) the group migrates to the para position with respect to the initial
occupied position. With temperatures of around 1600C, the ortho isomer is formed. This
rearrangement is known as Fries rearrangement.
OH
O
OH OCCH3 AlCl3
(CH3CO)2O 250C CH3
COCH3
CH3 CH3 AlCl3
OH
1600C H3COC

CH3

6.3.4 Kolbe and coupling reactions of phenol


Phenol is converted to ortho-hydroxybenzoic acid when it is treated with sodium
hydroxide and later with carbon dioxide.

OH ONa OH OH
NaOH CO2 H+

1250C, 4-7 Atm CO2H


CO2Na

Phenols couple with diazonium salt to form azo compounds. They are colored
compounds that are used as dyestuff pigments.

OH +
+ N2 HO N N

p-Hydroxyazobenzene
63
6.4 Uses of Phenols
1. Phenol itself is used as antiseptic (antimicrobiol). Modern antiseptics still contain
phenolic groups.
2. Phenol formaldehyde resin
3. Manufacture of dyes
4. Additive for odor and flavorings

OH OH
Cl Cl OH

HO CH2(CH2)4CH3
Cl Cl
Cl Cl
Hexachlorophene n-Hexylresorciol

Phenolic Antiseptics

Some naturally occurring phenols have been used for a long time in food industries
because of their aroma and taste.

6.5. Summary
Phenols react just like aliphatic alcohols.
Phenol undergoes EAS reactions and is an ortho-para director.
Phenol is a precursor for many industrially useful compounds such as aspirin. This makes
it one of the most important derivatives of benzene.

6.6 Questions and solutions

Questions

Q1. By use of resonance structures and reaction mechanisms, explain why presence of
an electron withdrawing group at ortho and para positions increase the acidic
character of phenols.

Q2. Outline synthesis of 2,6-Dichlorophenol from phenol

Q3. Suggest the major organic compounds (A-C) in the following transformations.

64
OH

(i) 1). CHCl3, OH-


A
2). H2O/H+
OH
NO2
(ii) H2SO4, H2O
B
Heat
SO3H

OH
NH2
(iii) 1). NaNO2, HCl
C
2). CuCl, Heat
Br

Solutions

1. See page 59

2.

OH OH OH
Conc. H2SO4 Cl Cl OH
Cl2
1). dil. H2SO4 Cl Cl
1000C FeCl3 Heat
SO3H SO3H 2). NaOH

3.
OH
OH
(i) NO2
CHO (ii)

OH
Cl
(iii)

Br

65
CHAPTER 7

ANILINES

7.0 Introduction
In chapter 6, phenols were discussed. It was noted that phenols are important
intermediates in organic synthesis. In this chapter, anilines will be dealt with. These are
compounds with an amino group attached directly to benzene. Most of its properties are
similar to those of aliphatic amines including its smell. However presence of a benzene
ring makes it different in some ways.

Objectives

By the end of this lesson, you should be able to:

• Explain the physical and chemical properties of anilines.


• Explain using equations different reactions shown by aniline.
• Describe how diazonium ion is prepared from various starting organic
compounds.
• Describe reactions of diazonium ions.
• State some of the uses of anilines and their derivatives.

7.1 Preparation and properties of aniline


When nitrobenzene is reduced by metal-acid reduction (Zn and HCl) aniline is formed.
Reduction with sodium borohydride can also be carried out but in presence a catalyst
poison Pd/C. They can also be prepared by heating azo compounds. Derivatized
halobenzenes can also be a source of anilines formed through nucleophilic aromatic
substitution reactions.

NO2 NH2
Sn/HCl
(Reduction)
NO2 NH2
NaBH4
Pd/C
Cl NH2
O2N O2N
NO2 NO2
NH3

NO2 NO2

66
In certain situations, selective and special reagents may be required to bring about the
reduction of the nitro group to amino group, depending on the type of group(s) attached
to the benzene ring. A few examples are cited below.

CH3 CH3
NO2 NH2
Sn/HCl
CH3
NH4HS
NO2 NO2 NO2
(NH3/HS)

NO2 NH2
NH2
Ni/H2
Method used when the
200-4000C compound has an acid
NHCOCH3 sensitive group
NHCOCH3

CHO CHO
SnCl2/HCl Method used when the
substituent has active
unsaturation.
NO2 NH2

Aniline has the following features:


1. Aniline is less basic than alkylamines, due to resonance delocalisation of the
lone pair of electron of nitrogen.
2. Basicity is enhanced by presence of electron donating group at ortho and
para. Electron withdrawing groups make aniline less basic than aniline itself
3. Amino group (NH2) is a strong activator. Presence of alkyl groups attached to
the ring reduces the activating power due to steric factors.
4. Electron withdrawing groups attached to nitrogen reduce the activating
property. A good example is an amide group. The lone pair of nitrogen is not
always available to activate the benzene ring, as it can also resonate with the
double bond of the carbonyl. Nitrogen with a quaternary system (-NR3), like
protonation of amino group, is a fully deactivating group.

+
NHCCH3 NH=CCH3
O O-

67
7.2 Reactions of aniline
Aniline can undergo two types of reactions; those that involve the amino group and those
that involve the benzene ring.

7.2.1 Reactions that involve the amino group

K NH-K+

NH2 +
NR3X-
3RX n-quaternary salt

+
1. CHCl3 N CH

2. 3OH- an isocyanide

NH2 CHO
N=CH H2O/H+ NH-CH2
+

Schiff's base (Imine)

NH2 N2+
NaNO2

H+/00C
Diazonium ion
NH2
NHCOCH3
(CH3CO)2O
CH3COONa

68
7.2.2 Reactions involving the benzene ring
Aniline is too reactive for the halogenation reactions that polyhalogenation takes place at
the para and two ortho positions. To introduce only one group, the amino group may be
acetylated first.

NH2
NH2 Br Br
H2O
+ 3Br2 2,4,6-Tribromoaniline

Br
NHCOCH3 NHCOCH3
NH2 NHCOCH3
(CH3CO)2O Br2 Br
+
CH3COOH
Br
H2O 80% 20%
NH2 H+

Br

The ortho isomer can be prepared by introduction at the para position first. Meta
substituted aniline product can be prepared from nitrobenzene. Iodo group to aniline
occurs at para position due to selectivity principle due to steric factor.

NH2
NH2 NHCOCH3
H2SO4 (CH3CO)2O

HO3S HO3S

Br2
H2O
NH2
H2O/H+ NHCOCH3
Br 100 C 0
HO3S Br
NH2 I2 NH2

NaHCO3/H2O
I
NO2 NO2 NH2
Br2
Sn/HCl
Lewis acid
Br Br

69
7.3 Diazonium salts
One of the most important reactions of anilines is the formation of diazonium salts. In
turn diazonium ion can be converted to a variety of organic compound, which otherwise
would require many steps to achieve from other aromatic compounds.

NH2 N2+ Cl-


NaNO2
+ NaCl + H2O
2HCl / 00C
Diazonium salt

Diazonium salt undergoes two types of reactions;

1. Replacement of N2 (loss of N2)


2. Coupling reactions where N2 is retained

7.3.1 Replacement of N2 (loss of N2)


Various nucleophiles can be used to replace the N2 group of the diazonium salt. Some
replacements can be intermediate to other important aromatic compounds. Direct
halogenation with fluorine is not possible but can be carried out from the diazonium salt
reacted with fluoroboric acid (HBF4). Replacement of the N2 group can be carried out by
use of hypophosphous acid (H3PO2) at low temperatures.

N2+
:Z Z
+ N2

Diazonium ion
CuCl
ArCl

CuBr
ArBr
ArN2+

Diazonium ion CuCN ArCN

KI ArI

70
Cl
CuCl CH3

CN CO2H
CuCN CH3
NH2 N2+Cl-
CH3 H2O/H+
CH3 NaNO CH3
2

HCl / 00C N2+BF4- F


HBF4
CH3
Heat
CH3

NaOH OH

CH3

H3PO2 CH3
+ H3PO3
H2O
0-250C

7.3.2 Coupling reactions where N2 is retained


Diazonium salts undergo coupling reactions to form azo compounds. They are strongly
colored compounds that have been used for a long time as dyestuffs. The aromatic
compound that is undergoing attachment must have a powerful electron-releasing group
like -OH, -NR2, -NHR and -NH2.

N=N
+
N(CH3)2 + N2 SO3-Na+
(H3C)2N SO3-Na+
Methyl orange
Red in acidic media
Yellow in basic media

OH
+
+ N2 NO2 OH
N=N
'Para Red' Dye

NO2

71
7.4 Summary
Aniline is less basic than alkyl amines due to resonance delocalisation of lone pair of
electrons on nitrogen.
Aniline shows reaction both at the amino group and at the benzene ring.
Diazonium ion can be converted to many organic compounds which otherwise would
require many steps. This makes diazonium ion very important in organic synthesis.

7.5 Questions and solutions


Q1. By use of resonance structures and reaction mechanism, explain the following.

(i) Introduction of an acetyl group (COCH3) to nitrogen reduces the activating


properties of aniline.

Q2. Suggest the major organic compounds (A-F) in the following transformations.

+
OH N N Cl-

(i) + A

CH3

Cl2/H2O
(ii) NH2 B

NH4HS
(iii) O2N CH3 C
(NH3/H2S)
NO2

Q3. Outline synthesis of the following compounds.

(i) 2,6-Dichlorophenol from phenol


(ii) o-Nitroaniline from aniline
(iii) m-chlorophenol from benzene

Solutions

1. (a) See page 67

72
2.

OH Cl
(i) N=N (ii) Cl NH2

Cl
CH3

(iii) H2N CH3

NO2

3.

NH2 NHCOCH3 NHCOCH3 NHCOCH3


NH2
HNO3 NO2
(CH3CO)2O H2SO4
(i) H2O/H2SO4 NO2
H2SO4 0
100 C
SO3H SO3H
NO2 NO2 NH2
(ii) HNO3 Cl2/FeCl3 Sn/HCl
H2SO4
OH-
Cl Cl
NaNO2/H+
OH N2+Cl-
HCl/ 00C
NaOH
Cl Cl

73
CHAPTER 8

POLYNUCLEAR AROMATIC COMPOUNDS: NAPHTHALENE

8.0 Introduction
In chapters six and seven, we looked at phenols and anilines and their derivatives and
their importance as precursors for synthesis of many and useful organic compounds. In
this chapter will focus mainly on EAS reactions of naphthalenes, di-nuclear aromatic
compounds and their derivatives. Orientation patterns of substituents on these compounds
as well as their importance in organic synthesis will also be discussed.

Objectives

By the end of this lesson, you should be able to:

• Describe steps involved in synthesis of naphthalene from benzene


• Write down oxidation and reduction products of naphthalene
• Describe using mechanisms, EAS orientation patterns of naphthalenes
• Predict EAS products of naphthalenes with various nucleophiles
• State the effect on the rate of reaction and of a substituent on naphthalene on
incoming nucleophile
• Discuss the reactions of substitute naphthalenes

8.1 Synthesis of Naphthalene


Naphthalene has two benzene rings fused together. Naphthalene main source is coal tar.
It has resonance energy of 61 Kcalmol-1. As has already been mentioned in chapter 2 that
naphthalene has three carbon centers that are not equivalent. It is therefore expected to
give more products during chemical reactions.

This is a multiple steps synthetic route. It involves electrophilic substitution of benzene.


This step works well if benzene is derivatized with an alkyl or halide groups. Second step
involves ring closure while the third step is for aromatization of the extra-formed ring.

74
O O O
HO HO
X X X
AlCl3 Zn/Hg/HCl
+ O
Clemmensen
O Reduction HF
O
or
H3PO4
O
X X -
Heat/Pd N2H4/OH X
Heat
Wolff-Krishner
Reduction

8.2 Reduction and oxidation reactions of naphthalene


Naphthalene can undergo oxidation and reduction reactions to give a variety of products
depending on the reagent used.

8.2.1 Reduction Reactions


When naphthalene is refluxed with ethanol in presence of sodium at 780C, the major
product is 1,4-dihydronaphthalene. Addition of four hydrogens is achieved when it is
refluxed with pentanol in presence of sodium at 1320C to form 1,2,3,4-
tetrahydronaphthalene. However, hydrogen in presence of metal catalyst nickel, platinum
or palladium, naphthalene is fully reduced to decahydrodecalin.

Na/CH3CH2OH
1,4-Dihydro
Reflux (780C) naphthalene

Na/C5H11OH 1,2,3,4-Tetrahydro
0 naphthalene
Reflux (132 C)

H2 Decahydrodecalin
Ni or Pt or Pd

The mechanism of formation of 1,4-dihydronaphthalene follows the Birch reduction. The


intermediate involved has the resulting negative charge and the radical are far apart and
thus more stable. The alcohol supplies the hydrogens involved in the reaction.

75
- -
Na. .
EtOH
(-Na+) (-EtO-)
. .
More stable
Na.
(-Na+)

EtOH
(-EtO-)
-

8.2.2 Oxidation of Naphthalene


In presence of chromium trioxide at room temperature, naphthalene is oxidized to 1,4-
naphthoquinone. However, oxidation in presence of vanadium pentoxide (V2O5) at high
temperatures, it is converted to phthalic anhydride.

CrO3/HOAc
1,4-Naphthoquinone
250C (40%)
O

O
V2O5/O2 Phthalic anhydride
0
O (76%)
460-480 C
O

8.3 Orientation and EAS reactions of naphthalene


Naphthalene is more reactive than benzene towards electrophilic aromatic substitution
reactions. Electrophilic attack on naphthalene can occur at two positions, α or β.
Orientation is controlled by the stability of the resonance structures involved as
intermediates. If the aromaticity is destroyed in both rings, then that is unfavorable, and
that route may not be followed. Normally the α substitution is preferred over the β
substitution.

It is noted that a β attack leads to formation of more resonance structures where the
aromaticity is lost in both rings unlike α attack with more stable resonance arrangements.

76
E H E H E H
+
E +
α attack
+ +
Stable Stable Less stable

E H
+

Less stable

+ E + E E
E+
H H H
β attack +
Stable Less stable Less stable
E

H
+
Less stable

8.4 Electrophilic aromatic substitution reactions of naphthalene


Most of the electrophilic aromatic substitution reactions of naphthalene are similar to
those of benzene except for the fact that there are two different sites in naphthalene. For
example, nitration and halogenation reactions occur position. Halogenation does not
require the Lewis acid catalyst. Nitro group can be reduced to naphthalmine and in turn
be converted to diazonium salt of naphthalene, which can lead to a variety of products as
had been indicated. Bromonaphthalene, for example, can be converted to a Grignard
reagent by reacting it with magnesium. This in turn can be converted to various
compounds.

Sulfonation of naphthalene, on the other hand, depends on the reaction temperature. At


lower temperatures 1-naphthalene sulphonic acid, the kinetic product is formed. At high
temperature, however, the major product is 2- naphthalene sulphonic acid, the
thermodynamic product. Heating the kinetic product forms the thermodynamic product.

77
NO2
HNO3
H2SO4 1-Nitronaphthalene
(90-95%)

Br

Br2/CCl4 1-Bromonaphthalene
Reflux (75%)

Mg
MgBr

Grignard Alcohols
Reagent Ketones
acids etc

SO3H
Conc H2SO4
1-Naphthalene
800C sulphonic acid
(Kinetic product)

1600C

Conc H2SO4 SO3H


2-Naphthalene
sulphonic acid
1600C (Thermodynatic
product)

In Friedel Crafts acylation the solvent used determines the products. When the solvent is
tetrachloroethane (CH2Cl4), 1-acetonaphthalene is formed while use of nitrobenzene as
the solvent leads to formation of 2-acetonaphthalene.

COCH3

CH2Cl4 1-Acetonaphthalene
(93%)
CH3COCl
AlCl3 COCH3
C6H5NO2
2-Acetonaphthalene
(90%)

The acetyl group can be converted to a carboxyl group reacting the aceto product with
bromine in a basic medium and later hydrolyzing in acidic medium. Alternatively, the
process can be achieved by use of sodium oxochlorous (NaOCl) at 60-700C.

78
COCH3 CO2H
-
1. Br2/OH
2. H+ + CHBr3

1-Naphalene
carboxylic acid

COCH3 CO2H
NaOCl
+ CHCl3
60-700C
1-Naphthalene
carboxylic acid

8.5 Orientation of electrophilic substitution in naphthalene derivatives


Introduction of a second substitution in naphthalene is more complicated than for
benzene. The incoming group may be attached to any of the rings. However the following
rules may assist in predicting where the substituent goes:

a. An activating group on the ring tends to direct further substitution on the same
ring. For example, the group is at position 1 (α) it direct the incoming group to
position 4 more than position 2. If the group is at position 2 (β), the incoming
group is directed more to position 1.
b. A deactivating group tends to direct further substitution on the other ring. If at
position 1 (α) a nitro group will direct the electrophile to α or β of the other ring.
Sulphonation depends on the reaction temperature.

Some sample reactions are presented below.

OH N2+Cl- OH
NaOH
+ 4-Phenylazo-1-naphthol
0-100C

N=N

OH
OH
NO2
H2SO4
+ HNO3
200C 2,4-Dinitro-1-naphthol
NO2

79
N2+Cl-
OH NaOH
+ 0-100C OH
N=N 1-Phenylazo-2-naphthol
Br
CH3 CH3
Br2 in dark

1-Bromo-2-methyl
naphthalene

NO2 NO2 NO2 NO2


HNO3
+
H2SO4/ 00C
NO2
Minor Major

NO2 NO2
H2SO4

SO3/ High temp


HO3S

8.6 Summary
Naphthalene can be reduced to various derivatives depending on the reagents and
conditions used.
Naphthalene is more reactive to EAS reactions than benzene and has two different
positions (α and β).
An activator on naphthalene directs the second substituent to the same ring while a
deactivator directs it to the other ring.

80
8.7 Questions and Solutions

Questions

Q1. Suggest the major organic compounds (A-C) in the following transformations.

OH
Br2 CH3COCl
(i) A (ii) B
AlCl3
OCH3

NH2

H2SO4
(iii) C
2000C/14 atm

Q2. Using a reaction scheme (step by step), show how the following conversions can
be achieved.

CO2H

(i)

Q3. By use of resonance structures and reaction mechanism, explain why 1-


Bromonaphthalene is exclusively formed and not 2-bromonaphthalene in
monobromination of naphthalene

Solutions
1.
Br COCH3

OH
(i) A= (ii) B=

OCH3
OH

(iii) C=

81
2.
NO2 NH2 N2+Cl-
HNO3 Sn/HCl NaNO2/HCl
-
OH 00C

CuCN
COOH CN

H+/H2O

3. (a) See page 81-82

82
CHAPTER 9

ANTHRACENES AND PHENANTHRENES

9.0 Introduction
In chapters 8 we looked at naphthalenes and their derivatives. In this chapter will focus
mainly on anthracenes and phenanthrenes. They are tri-nuclear aromatic compounds and
their derivatives. Orientation patterns of substituents on these compounds as well as their
importance in organic synthesis will also be discussed.

Objectives

By the end of this lesson, you should be able to:

• Differentiate anthracenes and phenanthrenes in terms of structural features.


• Write down oxidation, reduction and Diels-Alder products of anthracene
• Describe EAS reactions of anthracenes and phenanthrenes.
• Describe positional activity of anthracenes and hence predict EAS products of
anthracene with various nucleophiles.
• State some of the important derivatives of anthracenes.

9.1 Anthracene: Oxidation, reduction and Diels-Alder reactions


These are aromatic compounds with three rings fused together but differently. In
anthracene the rings are linear while as in phenanthrene the fusion is tilted to one side.
The resonance energy of anthracene is 84 Kcalmol-1 while that of phenanthrene is 92
Kcalmol-1.

8 9 1 9 10
7 2 8 1
3 2
6 7
5 10 4
6 5 4 3
Anthracene
Phenanthrene

Anthracene can be prepared from phthalic anhydride and benzene in presence of Lewis
acid.

83
O O O
AlCl3 H2SO4
O +

O O OH
O
Phthalic anhydride
Zn

Heat

Anthracene and phenanthrene undergo similar reactions. Most of their reactions involve
the middle ring.

9.1.1 Oxidation and reduction reactions


Oxidation of anthracene and phenanthrene with potassium dichromate in presence of an
acid give 9,10-anthraquinone and 9,10-phenanthraquinone, respectively. Reduction
involves the same position, where anthracene forms 9,10-dihydroanthracene while
phenanthrene is reduced to 9,10-dihydrophenanthrene.

O
K2Cr2O7
9,10-anthraquinone
Heat
O
K2Cr2O7
Heat 9,10-phenanthra-
quinone

O O
Na/EtOH
9,10-Dihydro-
antracene

Na/EtOH 9,10-Dihydro-
phenanthrene

9.1.2 Diels Alder reaction


Anthracene is able to undergo Diels Alder reaction with maleic acid while phenanthrene
does not. This reaction is used to differentiate the two compounds.

84
O
O

O O

+ O

O
Maleic acid
anhydride

9.2 Halogenation of anthracene


Anthracene reacts with different chlorinating agents to give various chlorinated products.
Chlorine gas in presence of sulphur carbide adds the two chlorine atoms with aromaticity
in the middle ring being lost. However aromaticity is restored if the product is reacted
with a base at high temperatures. Copper (I) chloride in presence of sulphur carbide
introduces only one chlorine atom by substitution. On the other hand sulphonyl chloride
(SO2Cl2) introduces two chlorine atoms by substitution.

Cl
Cl2/CS2 OH-
Heat

Cl Cl

CuCl
CS2
Cl
Cl
SO2Cl2

Cl

9.3 Sulphonation of anthracene


Sulphonation is one set of reactions where the reaction involves either one or the two
outer rings. Anthracene reacts with sulphuric acid in presence of glacial acetic acid to
give two products at different proportions depending on the reaction temperature. At low
temperatures 1-anthracenesulphonic acid is favored, while at high temperatures 2-
anthracene is formed at higher proportion. Excess sulphuric acid in absence of glacial
acetic acid also yields two disubstituted products, where the amount of each depends on

85
reaction temperature. Low temperature favors formation of 1,8-anthracene disulphonic
acid while high temperatures favor the 2,7-analogue.

SO3H
SO3H
H2SO4
+
Glacial CH3CO2H
1- 2-
Favored at low Favored at high
temperature temperature

SO3H SO3H

Excess H2SO4
+ SO3H

HO3S
1,8- 2,7-
Favored at low Favored at high
temp. Temp.

9.4 Alcohols of anthracene


In anthracene and phenanthrene, there are three different substitutable hydrogens.
Therefore there are three different alcohols; 1-anthrol, 2-anthrol and 9-anthrol. These
three alcohols have different physical properties as seen below.

OH OH
OH

1-anthrol 2-anthrol 9-anthrol


Yellow solid Brown solid Yellow solid
0 0 0
mp 200 C mp 152 C mp 120 C

9.5 Positional activity of anthracene


Electrophilic substitution reactions of anthracene, as has been mention earlier, can take
place in three different positions, 1, 2 or 9. The stability of the resonance structures
involved determines the product.

From the resonance structures below, it can be deduced that substitution at position 2
least preferred. Position one ideally would be preferred over position 9 (or 10) because in
1 the intermediate resonance structures preserve naphthalene, which is more stable with
61 Kcalmol-1 as compared to 9 (or 10) where its benzene with 36 Kcalmol-1 is preserved.

86
E E
+
E+
Position 9
or 10 +
Aromaticity in
2 rings preserved
E E
+ +
E
Position 1
aromaticity preserved +
in 2 rings both resonance
structures
+ E +
E+ E

Position 2
Aromaticity of 2 rings
destroyed
9.6 Anthraquinones
These are most important derivatives of anthracene. They are colored compound that are
used as dyestuffs. Many of them occur naturally are involved the coloration of some plant
materials.

In the laboratory, anthraquinone is prepared from phthalic anhydride and benzene in


presence of a Lewis acid. However, commercially, anthraquinone is prepared from
anthracene by oxidation with sodium dichromate in presence of sulphuric acid.

O O O

AlCl3 H2SO4
O +
(-H2O)
O CO2H
O
Phthalic anhydride Anthraquinone
O (90-95%)
Na2Cr2O7

H2SO4
Anthracene O

The most interesting reactions involving anthraquinones are the reduction reactions.
When it is treated with tin and hydrochloric acid in presence of acetic acid, anthrone is
formed. When heated in presence of zinc and hydrochloric acid a bi-anthracene, biathryl

87
is obtained. However, zinc in presence of a base leads to formation of 9,10-dihydro-9-
anthrol.

O
Sn/HCl
CH3CO2H Anthrone

O Zn/HCl

Bianthryl

Zn/OH-/H2O 9,10-Dihydro-
9-anthrol
OH

9.7 Summary
Anthracene undergoes similar EAS reactions as naphthalene.
Substituent at position 1 on anthracene is preferred over 2 and 9 because the intermediate
resonance structures preserve naphthalene. Substitution at position 2 is least preferred.

9.8 Questions and solutions


Questions

Q1. Suggest the major organic compounds (A-C) in the following transformations.

(i) SO2Cl2
A (ii) HNO3
B
CH3CO2H

1). NaCrO7/H+
(iii) C
2). Sn/HCl
CH3CO2H

Q2 By use of resonance structures and reaction mechanism, explain the why


Aromatic electrophilic substitution of anthracenes is preferred at position 9 over
position 2.

88
Solutions
1.
Cl

(i) A= B=
(ii)
NO2
Cl
NO2

(iii) C=

2. See pages 91-92

89
REFERENCES
Adams, R., Johnsons; J., and Wilcox, C. (1968). Laboratory Experiments; 5th Edition,
Macmillan Publing Company, New York.

Brian, S. Furniss D. (1989). Vogels Text Book of Practical Organic Chemistry; 4th
Edition.

Bruice, P.Y. (1995). Organic chemistry; 1st Edition, Hall Inc, New Jersey.

Clayden, Greeves, Warren and Wothers. (2001). Organic Chemistry; 1st Edition, Oxford
University Press, New York.

Fessenden, R.J and Fessenden, J.S. (1990). Organic Chemistry; 4th edition, Cole
Publishing Company, Belmond.

Finar, I.L (1973). Organic Chemistry; 6th edition. Longman, London.

Hart, H., Craine, L.E. and Hart, D.J. (2003). Organic Chemistry –A short course. 11th
Edition, Houghton Mifflin Company.

Morrison, R.T and Boyd, R. N. (1987). Organic Chemistry; 5th edition, Longman, New
York.

Solomons,T.W.G. (1996). Organic Chemistry; 6th Edition, John Wily and Sons Inc, New
York.

Sykes, P. (1981). A guide to Mechanisms in Organic Chemistry. 5th Edition, Longman,


London.

90
PRACTICALS

ELECTROPHILIC AROMATIC SUBSTITUTION

Introduction
Most electrophilic substitution reactions of aromatic compounds are thought to take place
according to the following general mechanism:-
Using benzene as an example:

H E
H E
Slow Fast
+ E+ + + H+

In some cases, the experimental evidence is interpreted best by proposing that E+, the
electrophile is completely formed as a cation before it reacts with the aromatic compound
as implied in the above equation.

In other cases, the data are best interpreted in terms of simultaneous attack of aromatic
system on E-X and loss of the leaving group

H E
H E
Slow Fast
+ E-X + + X- + H+

For convenience however, electrophilic aromatic substitution reactions are often


interpreted in terms of attack by previously formed E+, with the mental reservation that
this may not always be the best representation of reaction
The experiments that follow have been selected to illustrate some of the important
aspects of electrophilic aromatic substitution reactions (EAS)

91
EXPERIMENT 1: PREPARATION OF NITROBENZENE

Introduction
The conditions used in the nitration of an aromatic compound depend on the structure of
the compound. The nitrating agent in most cases appear to be nitronium ion:-

+
O N O

The source of nitronium ion may be a nitronium salt, such as nitronium


fluoroborateNO2+BF4- or as in this experiment, the ion may be derived from nitric acid by
reaction with conc. sulphuric acid.
+
i) HNO3 + H2SO4 NO2 + H2O + HSO4-
+
ii) NO2 + C6H6 C6H5NO2 + H+

Overall reaction: C6H6 + HNO3 C6H5NO2 + H2O


Nitrobenzene

Procedure
Before you do the experiment, note the following:

• The benzene and acid layers are only slightly soluble in one another. When
benzene dissolves in the acid layer, it reacts and swirling speeds the process of
solution.
• Nitrobenzene is appreciably toxic and its vapour should not be allowed to escape
in the atmosphere (keep the experiment in the fume hood). The liquid is also toxic
by skin absorption and should be washed if spilled on the body with methylated
spirit followed by soap and warm water.
• Benzene is toxic: Avoid excessive contact with either liquid or vapour, do the
experiment in the fume hood.

Place 8ml of conc. Nitric acid in 50 ml flask and add slowly 9 ml of conc. Sulphuric acid,
swirling gently and cooling the flask under running tap water. Then introduce, 7 ml
(0.075 mole) of benzene in small portions from a measuring cylinder. By constant
swirling and brief cooling periods in cold water, keep the temperature of the mixture
close to but not exceeding 60oC and not below 55oC, during the addition of benzene.

When all the benzene has been added, attach a reflux condenser and heat the mixture on a
water bath at 60oC for 30 minutes. From the time to time during this period lift the flask
and condenser out of the water bath, and swirl the reaction mixture vigorously to ensure
good mixing of the immiscible layers. At the end of refluxing, cool the flask in cold

92
water. Pour the complete reaction mixture into about 60 ml of cold water in a beaker (200
ml), and swirl the mixture well order to wash out as much acid as possible from
nitrobenzene and allow to stand. When nitrobenzene has settled to the bottom, pour off
the acid liquors completely as possible and transfer the residual liquid to a separatory
funnel. Add 10 ml of 1.5 M sodium carbonate solution, shake gently, frequently releasing
the pressure. Allow to stand and then remove the upper aqueous layer. Repeat the process
with two further 10 ml portions of sodium carbonate solution, followed once by 10 ml
portion of water. Remove the upper aqueous layer in each case and retain the lower

Transfer the nitrobenzene to a stoppered test-tube or small conical flask and add a few
small lumps of anhydrous calcium chloride, swirling occasionally until the solution is
clear.

Set up the apparatus for distillation using air condenser and distil the nitrobenzene,
collecting the fraction boiling at 198-202oC. Do not distil to dryness nor allow the
temperature to rise above 205oC, for there may be a residue of m-dinitrobenzene and
higher nitro compounds and an explosion may occur.

Record your yield and determine the refractive index of the nitrobenzene

Questions
1. Outline, using equations production an electrophile using conc. Nitric and
sulphuric acids.

2. Using appropriate structural formula, indicate the mechanism of nitration of


benzene.

3. Nitration by nitric acid alone is believed to proceed by essentially the same


mechanism as nitration in the presence of sulphuric acid. Write the equation for
the generation of nitronium ion from nitric acid alone.

4. Explain why nitrations are usually carried out at comparatively low temperatures.

93
EXPERIMENT 2: Reduction of nitrobenzene to Aniline

Introduction
Aromatic nitro compounds may be reduced to the corresponding aromatic amines by
many reagents. In this experiment, nitrobenzene is reduced to aniline by nascent
hydrogen, which is generated from tin and conc. hydrochloric acid.

C6H5NO2 + 6 H C6H5NH2 + 2H2O

This reaction is of particular importance as a step in the conversion of benzene to a very


large range of useful compounds that can be prepared from aniline.

Procedure
[Note: Ensure each joint is smeared with small amount of lubricant preferably silicone
grease in order to prevent seizure of the joints when using strongly alkaline solution]

Place 3 ml (0.03 mole) of nitrobenzene and 7 ml of granulated tin (small pieces) in 100
ml conical flask. Measure 15 ml of conc. Hydrochloric acid. Pour about 4 ml of the acid
down the condenser, and steadily swirl the contents of the flask. The mixture becomes
warm and before long the reaction should be quite vigorous; if it boils vigorously,
moderate the reduction somewhat by momentarily immersing the flask in cold water.
When the initial reaction slackens on its own, add another few mls of acid; continue to
swirl the flask, cool again if the reaction becomes too violent. Do not cool more than is
necessary to keep the reaction under control; keep the reaction mixture well mixed.
Proceed in this way until all the acid has been added. When all the acid has been added
and the vapour of the reaction has subsided, heat the flask in a boiling water bath for
about 30 minutes. This completes the reduction of nitrobenzene to aniline by stannous
chloride. If after 30 minute heating, some nitrobenzene still remains, as is evidenced by
oily droplets on the surface of the reaction mixture and by the characteristic smell of nitro
compounds, add a few more of small pieces of tin and 2-3 ml of conc. Hydrochloric acid
and continue heating on the water bath until a homogenous solution is obtained. The
aniline is now present as complex salt, aniline chlorostannate, which although
appreciably water- soluble may separate during the reduction, particularly during the
cooling which follows.

Cool the reaction mixture to room temperature. In a 100 ml conical flask, make up a
solution of sodium hydroxide (11.5 g in 15 ml water) and cool it to room temperature
also.

94
Add the sodium hydroxide gradually and with gentle swirling to the cooled reaction
mixture. If the resulting mixture boils during the addition of the alkali, cool again. Keep
the reaction mixture in ice-cold water for about 5 minutes.

The aniline, which is now present as the free base, will now be visible as a distinct upper
layer. Decant off, from any residue solid, both the aqueous and aniline layer in 50 ml
quick fit flask. Equip the flask for steam distillation using Claisen still head. Steam distil
by heating the flask directly until no more drops of aniline come over. Collect the
distillate in a separating funnel

Add 3 g of sodium chloride to the separating and shake. This reduces the solubility of
aniline in water (about 3%). Run off the crude aniline into a test-tube. Add a few pellets
of sodium hydroxide to remove moisture, and allow to stand in a stoppered tube until
turbidity disappears.

Determine the refractive index of your aniline, a confirmation of its identity.

Questions
1. Outline with the aid of appropriate chemical equations, the various stages in the
reduction of nitrobenzene to aniline using tin and hydrochloric acid.

2. Give two alternative reagents that can be used to reduce nitrobenzene to aniline
other than metal-acid reagents.

3. Explain why calcium chloride cannot be used as a drying agent for aniline.

4. State the reagents you would use to convert 1-methyl-2,4-dinitrobenzene into the
following;

a) 2-amino-1-methyl-4-nitrobenzene.
b) 1-amino-4-methyl-2-nitrobenzene.

95
EXPERIMENT 3: The nitration of Aniline to p-nitroaniline

Introduction
The amino group activates benzene ring towards electrophilic reagents to a greater extent
than the hydroxyl group, and aniline is very readily oxidized by nitric acid, so direct
nitration is impossible. It is therefore necessary to protect the amino group while carrying
out the nitration. This is conveniently done by acetylation. The acetyl group does not
change the position, to which the substituent is directed, but it is fairly a large group and
hence, the yield is nearly all p-nitroacetanilide. The acetyl group is finally removed by
alkali hydrolysis according to the following equation
NH2 NHCOCH3 NHCOCH3 NHCOCH3

(CH3CO)2O HNO3 NaOH

CH3CO2H H2SO4
NO2 NO2

Procedure

Step 1: Preparation of acetanilide


Put 5 ml (0.05 mole) of aniline, 7 ml ethanoic anhydride (in excess) and 5 ml of
concentrated ethanoic acid into a 50 ml quick fit flask fitted with water condenser. Reflux
the mixture for 15 minutes.

Cool the mixture and then pour it with stirring into 50 ml of water in a 250 ml beaker.
The acetanilide will solidify into white lumps. Cool the mixture, filter off the solid at the
pump, wash it with water and finally dry it in the oven at 90-100oC (mpt of acetanilide is
114oC)

Step 2: Nitration of Acetanilide


Put 2.5 ml of conc. Ethanoic acid into a 100 ml conical flask and add with stirring 2.7 g
(0.02 mole) of acetanilide followed by 5 ml of conc. Sulphuric acid. Cool the hot clear
solution in a freezing mixture of ice and sodium chloride, stirring gently with a
thermometer until the temperature is between 0 and 5oC. Now, add drop wise with
stirring, 1.5 ml of fuming nitric acid, keeping the temperature below 25oC

96
When all the acid has been added and temperature begins to fall, remove the solution
from the freezing mixture and allow to stand for ten minutes. If the temperature starts to
rise steadily, return the solution to the freezing mixture for a while.

Now pour the solution into 50 ml of water contained in 250 ml conical flask. The p-
nitroacetanilide will precipitate as a yellow crystalline powder. Filter off at the pump,
wash with water and drain thoroughly

Step 3: Hydrolysis of p-nitroacetanilide


Transfer the crystals of p-nitroacetanilide to a 50 ml quickfit, add 8 ml of ethanol and a
solution of 1 g of NaOH in 5 ml of water. Reflux the mixture for about ten minutes, after
which there should be a clear yellow solution. Cool the flask to crystallize the p-nitro-
aniline

Filter at the pump, using the filtrate to wash out the last part of the solid from the flask,
and wash with a little water. Drain thoroughly, recrystallise from the minimum quantity
of aqueous Ethanol and dry the product. Determine the melting point of the pure p-
nitroaniline and calculate the molar yield based on the amount of acetanilide used.

Questions
1. Give an alternative reagent to the one you used in the above practical; for
protecting the amino-group during nitration.

2. If direct nitration of aniline is carried out in strongly acidic medium, a


considerable % of the amine is, as expected converted to a complex variety of
oxidation products. Such nitration as does occur however, results in m-
nitroaniline as the major product rather than para-ortho isomers that might have
been expected due to the presence of NH2 group. Explain.

3. Explain why -NH2 group activates the ring more than the -OH group.

4. Outline, giving reagents employed, how you would convert p-toluidine(4-


methylaniline) to 4-amino-3-bromotoluene.

5. Arrange the following functional groups in increasing order of their activation of


aromatic rings towards electrophilic reagents: -NHCOCH3, -NH2 -NHCH3,
-OH, -OCH3, -SH, -SCH3.

97
EXPERIMENT 4: Preparation of iodobenzene
Introduction
It is difficult to iodate benzene because the reaction between benzene and iodine is
reversible:-

C6H5NO2 + I2 C6H5I + HI

The most satisfactory way of attaching a halogen atom to an aromatic ring is by


replacement of the diazonium group, which is in turn obtained from the corresponding
primary amine.

Replacement of diazonium group by an iodine atom is readily effected by direct reaction


of iodine with the appropriate diazonium compound. Thus iodobenzene is usually
prepared by the reaction of benzene of diazonium chloride and potassium iodide.

[C6H5N2 ]+Cl- + KI C6H5I + KCl

Iodobenzene is purified from involatile inorganic impurities by steam distillation, as


iodobenzene is almost insoluble in water. To obtain a good separation, the distillate is
shaken with several portions of diethyl ether, as the partition coefficient for iodobenzene
favors the either layer. Final purification is by distillation.

Procedure

Step 1: Preparation of benzene diazonium chloride solution


Place 4 ml (0.4 mole) of aniline in a 100 ml flask, and add a mixture of 12 ml of conc.
HCl and 12 ml of water. Swirl gently to dissolve the aniline. Place a thermometer in the
flask and cool the mixture in an ice-salt mixture.

Dissolve 3.4 g of sodium nitrite (NaNO2) in 4 ml of water in a test-tube, and cool in the
freezing mixture. Introduce the mixture slowly into the flask. Stir the mixture with the
thermometer and do not allow the temperature to rise above 10oC. The flask now contains
a solution of benzene diazonium chloride.

Step 2: Preparation of iodobenzene

[Note: Ether is very flammable]

98
Dissolve10 g of potassium iodide in 12 ml of water in a test-tube and cool in the freezing
mixture. Add the solution to the benzene diazonium chloride solution, keeping the flask
in the freezing mixture not to allow the temperature to rise above 10oC. When all the KI
solution has been added, transfer the flask to a cold water-bath and fit a reflux condenser

After ten minutes, bring the bath to a boil and maintain it at this temperature for about 15
minutes. Nitrogen is evolved and dark oily drops of impure iodobenzene separates out.

Steam distil the mixture until no more oily drops appear in the distillate.

Extract the iodobenzene from the distillate with three successive 5 ml portions of diethyl
ether, using a separating funnel.. Combine the ethereal extracts and wash them with 5 ml
of 10% NaOH and then with 5 ml of water.

In each case reject the washings. Dry the ethereal solution of iodobenzene by allowing it
to stand in contact with little fused calcium chloride in a stoppered test-tube until the
solution is clear.

Distil off the ether from hot water bath. [Extinguish all flames in the lab]. Remove the
hot water-bath and distil the iodobenzene using the condenser as an air condenser. Collect
the fraction distilling between 178oC and 184oC.

Determine the refractive index of iodobenzene and calculate the molar yield based on the
aniline used.

Questions
1. Outline the synthesis from benzene diazonium chloride, the following
compounds:-

a) Chlorobenzene b) Flourobenzene c) Phenol

d) Benzoic acid e) Benzene

2. Outline the synthesis of m-nitrobenzene from toluene.

99
EXPERIMENT 5: Preparation of methyl orange (p-(p-dimethyl-
aminophenylazo) benzene sulphonic acid, sodium salt)

Introduction
Under the correct conditions, diazonium salts react with certain aromatic compounds to
yield products of the general formula Ar-N=N-Ar, called azo compounds. In this reaction
known as azo- coupling, the nitrogen of the diazo group is retained in the product, in
contrast to the majority of reactions involving diazonium salts

ArN2+ +Ar’H Ar-N=N-Ar’ + H+


Azo compound

The aromatic ring (Ar’H) undergoing attack by diazonium ion must, in general contain a
powerful electron releasing group because diazonium ion ArN2+ is only weakly
electrophilic and thus only capable of attacking very reactive rings. Coupling is a further
example of EAS, in which the substitution usually occurs para to the activating group
(R).

R R R

+ ArN2+ + + H+

H N=NAr
N=NAr

Azo compounds are strongly colored: are yellow, orange, red, blue or green depending on
the exact structure of the molecule. Hence their great importance as dyes. Color changes
also occur depending on the pH, thus their use as acid base indicators, for example
methyl orange.

Procedure

Step 1: Diazotisation of sulphanilic acid


In a 250 ml conical flask, dissolve by boiling 4.8 g of sulphanilic acid crystals in 50 ml
of 2.5% sodium carbonate solution. Cool the solution under the tap, add 1.9 g of sodium
nitrite and stir until it is dissolved. Pour the solution into the flask containing about 25 g
of ice and 5 ml of conc. HCl. In a minute or two, a powdery white precipitate of
diazonium salt should separate and is to be used in this form as its stable for a few hours.

100
Step 2: Preparation of methyl orange
In a test-tube, thoroughly mix 3.2 ml (0.025 mole) of dimethylaniline and 2.5 ml of
glacial ethanoic acid. To the suspension of diazotised sulphanilic acid contained in a 400
ml beaker add, with stirring the solution of dimethylaniline acetate. Wash out the test
tube with some of the diazotised sulphanilic acid solution to ensure complete transfer of
dimethylaniline acetate. Stir and mix well and within a few minutes, to red acid stable
form of the dye should separate.. a stiff paste should result in 5-10 minutes and 35 ml of
10% NaOH is then added to produce the orange salt.

Stir well and heat the mixture to boiling point, when a large part of the dye should
dissolve. Place the beaker in a pan of ice and water and let the solution cool undisturbed.
Collect the product on a buchner funnel using saturated NaCl rather than water to rinse
the flask and to wash the dark mother liquor from the filter cake.

Recrystallise the crude product from water, after making preliminary tests to determine
the optimum conditions.

Calculate the molar yield based on the dimethylaniline used.

Questions
1. Write equations for:

a) Diazotisation of sulphanilic acid

b) Conversion of diazotised sulphanilic acid to methyl orange

2. The efficiency of the coupling reaction between diazonium salts and other compounds
is very dependent upon reaction conditions and particular the pH. Explain why this
should be so.

3. An azo compound is readily cleaved at the azo linkage by tin (II) chloride to form two
amines. What is the structure of the azo compound that is cleaved to 4-amino-3-
bromotoluene and 4-amino-2-methylphenol?

101
EXPERIMENT 6: Preparation of p- Di-t-butyl benzene

Introduction
Introduction of alkyl groups into an aromatic ring is a useful process and is carried out on
an industrial scale to produce compounds such as cumene (1-mthylethylbenzene)

Alkylation involves attack of an electrophilic carbon species on the ring, choice of the
reagent and conditions depend on the alky group and the reactivity of the ring.
In this experiment, the major product of the reaction between benzene and t-butylchloride
is p-dibutylbenzene

C(CH3)3

+ AlCl3
2 (CH3)3CCl

C(CH3)3

Procedure
In a thoroughly dry 100 ml conical flask, place 10 ml of t-butylchloride and 5 ml (0.06
mole) of benzene. Arrange a trap for HCl gas by connecting a length of rubber tubing
from a one-hole rubber in the flask to a glass funnel inverted just below the surface of a
beaker containing water

Obtain 0.5 g of anhydrous aluminium chloride in a stoppered test tube. Minimize


exposure of this compound to air, the proper amount can be estimated by comparison
with weighed demonstration sample. Cool the benzene t-butyl chloride mixture in an ice
bath (clamp the flask loosely to avoid tipping). Add about one-third of the aluminium
chloride, and swirl the flask in ice bath. After bubbling has occurred for 4-5 minutes add
the rest of aluminium chloride in two separate portions during the next 10-15 minutes.
(restopper the flask as quickly as possible and rinse your fingers, since brief exposure to
the acid fumes cannot be avoided)

When the reaction begins to subside, remove the ice bath and allow the mixture to warm
to room temperature. Unstopper the flask and add 10 ml of ice-cold water. Then add
about 10 ml of diethylether (extinguish all the flames), swirl the mixture and transfer the
contents of the flask to a separating funnel; rinse the flask with small amount of ether and
add the washing to the separating funnel. Gently swirl the funnel for a few minutes taking
care to release the pressure frequent intervals. Separate the ether layer, wash it with two 5
ml portions of water ad dry it with a small amount of anhydrous magnesium sulphate in a
stoppered 50 ml quick fit flask.

102
Distil off the ether from hot water bath (put off all the fire in the lab). Using the same
flask, but with a reflux condenser added recrystallise the p-di-butylbenzene from 10 ml of
methanol, collect and air dry the product, record its yield and its melting point.

Calculate the molar yield based on the benzene used.

Questions.

1. State briefly any four limitations to the use of Friedel-Crafts alkylation.

2. GLC analysis of the crude reaction mixture from the experiment shows in addition to
unreacted benzene and p-di-butylbenzene, three other products. Suggest structures for
these by products and write equations for their formation.

3. Other than the comparison of melting points, how would you, using chemical method,
prove that the product you isolated in the experiment was p-di-butylbenzene?

103

You might also like